You are on page 1of 112

MATHEMATICS 9- LESSON 1

TOPIC: ILLUSTRATING QUADRATIC EQUATIONS

Quadratic Equation - a second-degree polynomial equation that can be


written in the standard form 𝑎𝑥 2 + 𝑏𝑥 + 𝑐 = 0 where 𝑎, 𝑏 𝑎𝑛𝑑 𝑐 are real numbers
and 𝑎 ≠ 0 .

Standard Form of quadratic equation : 𝑎𝑥 2 + 𝑏𝑥 + 𝑐 = 0

Solving Quadratic Equations by the Square Root Property


Illustrative Example: Solve x: 𝑥 2 − 16
Get all terms on the same side by the Addition 𝑥 2 − 16 = 0
Property of Equality.
(Different of two
squares)
By the Addition Property of Equality 𝑥 2 = 16
Extract the square root x = √𝟏𝟔 x = -√𝟏𝟔
x=4 x = -4
List all values of x. x = {-4, 4}

Solving Quadratic Equations by Factoring


Primarily three types of factoring

▪ Common Monomial Factor 𝑎𝑏 + 𝑎𝑐 = 𝑎 (𝑏 + 𝑐)


▪ Difference of Two Squares 𝑥 2 − 9 = (𝑥 + 3)(𝑥 − 3)
▪ Quadratic Trinomial 𝑥 2 − 5𝑥 + 6 = (𝑥 − 3)(𝑥 − 2)
Illustrative Example: Solve for x: 𝑥 2 + 3𝑥 = 0
Factor the common monomial. x(x + 3) = 0
Set each factor equal to 0 and solve for x x+3=0
x=0
x = -3
List all values of x. x = {0, -3}

1
Solving Quadratic Equations by the Quadratic Formula

Quadratic Formula-The solution of 𝑎𝑥 2 + 𝑏𝑥 + 𝑐 = 0, where a, b, and c


−𝑏±√𝑏 2 −4𝑎𝑐
are real numbers with 𝑎 ≠ 0 are 𝑥 = .
2𝑎

To use quadratic formula, first write the given equation in the form 𝑎𝑥 2 +
𝑏𝑥 + 𝑐 = 0, then identify the value of a, b and c substitute their values in the
quadratic formula.

Example:
Solve 𝑥 2 + 2𝑥 − 15 = 0.

Solution:
Given that 𝑥 2 + 3𝑥 − 15 = 0,
a=1, b=3, c=-15

−𝑏±√𝑏2 −4𝑎𝑐
𝑥=
2𝑎

−3±√32 −4(1)(−15)
𝑥=
2(1)

−3±√69
𝑥= 2

Thus,
−3+√69 −3−√69
𝑥1 = 𝑥2 =
2 2

2
Green Rose Center for Academe Inc.
Purok Gemelina Estaca Compostela Cebu
Mobile No. 0933-1617936 / Landline No. (032) 425-6216
E-mail add: greenroseacademe_119@yahoo.com, Website: http://grca.school
Government Recognition No:
PRE-ELEM: 04 S. 2017, ELEMENTARY: 5, S. 2017, JHS: 06, S.2017, SHS: 059 S.2018
School ID: 408281
MATHEMATICS 9- DRILL 1
(Topic: Illustrating Quadratic Equations)
Name: _____________________________Gr. & Sec. _________Date: ________
Parent’s Signature: ________________ Score: _________________
A. Directions: Determine if the given equation is quadratic or not. Write
your answer on the space being provided.

________________1. 3𝑥 + 𝑦 = 7
________________2. 𝑘 2 + 𝑘 + 15=0
________________3. 6𝑥 + 4𝑥 + 67 = 0
___________________4. 𝑥 3 + 1 = 0
___________________5. 5𝑥 2 + 2 = 7𝑥

B. Directions: Write the following in standard form and determine the


values of 𝑎, 𝑏, and 𝑐. Write your answer on the box being provided.

Given Answer

Standard Form a b C

1. 5𝑥 2 = 2𝑥 − 9

2. 3𝑥 2 − 7𝑥 = 4

3. 4𝑥 − 5𝑥 2 − 7

4. 2 = 𝑥 2 + 𝑥

5. 2𝑥 − 6 = 7𝑥 2

3
C. Directions: Solve the unknown problem. Write your solutions in a clean
sheet of paper.

1. Find the solution set of 𝑥 2 + 5𝑥 + 6 using factoring.


2. Solve the 𝑏2 = 81 by using extracting the square roots.
3. Find the solution set of 𝑚2 + 7𝑚 + 7 = 0 by using quadratic formula.
4. Solve the 2𝑥 2 + 5𝑥 = 1 by quadratic formula.
5. Given a value of 𝑘 so that 1 will be a solution to 5𝑥 2 − 1 = 𝑘(𝑥 + 1).

MATHEMATICS 9- ASSIGNMENT 1
(Topic: Illustrating Quadratic Equations)

Name: _____________________________Gr. & Sec. _________Date: ________


Parent’s Signature: ________________ Score: _________________

A. Directions: Explain the statement in your own words (atleast 5


sentences). Write your answer on the space being provided.

“In solving a quadratic equation, why is it important to make one side of


the equation equal to zero before factoring the other side?”

Criteria:
Content- 70%
Organization-20%
Punctuality- 10%

4
MATHEMATICS 9- LESSON 2
TOPIC: CHARACTERIZING THE DISCRIMINANTS

Discriminant – The discriminant (d) of a quadratic equation is used to


determine the nature of its roots. It is given by the formula 𝑏2 − 4𝑎𝑐 where 𝑎, 𝑏,
and 𝑐 are the coefficients of the quadratic equation in standard form 𝑎𝑥 2 + 𝑏𝑥 +
𝑐 = 0.
Example: Solve (x - 3) (x + 2) = 0
Step 1: Write in standard form. 𝑥2 − 𝑥 − 6 = 0
Step 2: Identify a, b, & c a = 1, b = -1, & c = -
6
= (−1)2 − 4(1)(−6)
Step 3: Solve for 𝑏2 − 4𝑎𝑐 =1
+ 24
= 25
Nature of the roots
The nature of the roots determined by the value of the discriminant can be
summarized in three cases:
1. If 𝑏2 − 4𝑎𝑐 = 0, the roots are one real roots.
2. If 𝑏2 − 4𝑎𝑐 > 0, the roots are two distinct real roots.
3. If 𝑏2 − 4𝑎𝑐 < 0, the roots are no real roots.
Example: 4𝑥 2 − 20𝑥 + 25 = 0
Solution: a = 4, b = -20, & c = 25
= (−20)2 − 4(4)(25)
=400-400
=0
Thus, there is one real roots.
Roots of Quadratic Equation
Roots - are called the x-intercepts or zeroes
To find the roots of a quadratic equation is to use the quadratic
formula.

5
Green Rose Center for Academe Inc.
Purok Gemelina Estaca Compostela Cebu
Mobile No. 0933-1617936 / Landline No. (032) 425-6216
E-mail add: greenroseacademe_119@yahoo.com, Website: http://grca.school
Government Recognition No:
PRE-ELEM: 04 S. 2017, ELEMENTARY: 5, S. 2017, JHS: 06, S.2017, SHS: 059 S.2018
School ID: 408281
MATHEMATICS 9- DRILL 2
(Topic: Characterizing The Discriminants)

Name: ___________________________Gr. & Sec. ___________Date: ________


Parent’s Signature: ________________ Score: _________________
A. Directions: Evaluate the discriminant of the following quadratic equations
and describe the nature of roots. Write your answer on the box being
provided.
Equation Discriminant Nature of Solution

𝑚2 + 2𝑚 + 3=0 1. 6.
2
5𝑚 + 𝑚 + 7=0 2. 7.
𝑚2 − 8𝑚 + 16=0 3. 8.
𝑚2 − 𝑚 − 2=0 4. 9.
2
2𝑚 − 9𝑚=0 5. 10.

B. Direction: Encircle the letter of correct answer.


1. What are the roots of the equation of 𝑥 2 − 4𝑥 + 3 = 0?
a. (-1, -3) b. (1, 3) c. (0, 3) d. (0, -3)
2. What are the roots of the equation of 𝑣 2 + 6𝑣 + 8 = 0?
a. (2, -4) b. (2, 4) c. (-2, -4) d. (0, 4)
3. What are the roots of the equation of 𝑘 2 − 7𝑘 + 6 = 0?
a. (6, 1) b. (1, 6) c. (1, -6) d. (2, 6)
4. What are the roots of the equation of 𝑏 2 + 4𝑏 − 12 = 0?
a. (6, 2) b. (-6, 2) c. (-6, -2) d. (3, 6)
5. What are the roots of the equation of 𝑥 2 + 3𝑥 = 0?
a.(-1, -3) b. (1, 3) c. (0, 3) d. (0, -3)

6
MATHEMATICS 9- LESSON 3
TOPIC: SOLVING THE ROOTS OF QUADRATIC EQUATIONS

Sum and Product of the Roots of a Quadratic Equation


If a quadratic equation is given in standard form, we can find the
sum and product of the roots using coefficient of x 2, x and constant term.

The sum and product of a quadratic equations are given by the formula:
𝑥1+ 𝑥2 = −𝑏⁄𝑎
𝑥1 (𝑥2 ) = 𝑐⁄𝑎

Example: Find the sum and product of the roots of 8𝑥 2 − 9𝑥 − 3 = 0.


Solution
Given 8𝑥 2 − 9𝑥 − 3 = 0
Step 1: Get the values of a, b, and a = 8, b = -9, c = -3
c.
Step 2: Find the sum of roots 𝑥1+ 𝑥2 = −𝑏⁄𝑎
= -(-9)/8
= 9/8

Step 3: Find the product of roots 𝑥1 (𝑥2 ) = 𝑐⁄𝑎


= -3/8

Write a quadratic equation with the given roots


How to Write a Quadratic Equation if the Roots are given?
When two roots of a quadratic equation are given, the formula to form
the quadratic equation is given by:
𝑥 2 - (sum of the roots) x + (product of the roots) = 0
𝒙𝟐 - (α+b) x +(ab)=o

7
Example: Construct a quadratic equation whose two roots are 7 and -1.
Step 1: Substitute the given roots. 𝑥 2 - (α+b) x +(ab)=o
𝑥 2 - (7-1)x + (7)(-1) = 0

Step 2: Eliminate the parenthesis. 𝑥 2 - 6x - 7 = 0

Thus, the quadratic equation of the given roots 7 and -1 is 𝒙𝟐 - 6x -


7 = 0.

Solving Equations Leading to Quadratics


Standard Form: 𝑎𝑥 2 + 𝑏𝑥 + 𝑐 = 0

Example: Transform the equation (𝑥 + 8)2 − 9𝑥 = 52 into quadratic


equation.
Step 1: Evaluate the exponent. (x+8)(x+8) - 9x = 52
Step 2: Eliminate the parenthesis by x(x)+x (8) +8(x)+8(8) - 9x =
multiplying the two binomials using the FOIL 52
METHOD. 𝑥 2 + 8x + 8x + 64 - 9x = 52

Step 3: Arrange the terms according to its 𝑥 2 + 8x + 8x - 9x + 64 - 52 = 0


highest degree of exponent and perform the 𝒙𝟐 + 7x +12 = 0
operations.

Thus, the answer of the given equation above is 𝒙𝟐 + 7x +12 = 0.


Step 4: To get the values of 𝒙𝟐 + 7x +12 𝑥 2 + 7x +12 = 0
= 0 use factoring method (x + 3)(x+4) = 0

Step 5: Get the factors equal to zero. x+3=0 x+4 = 0


x = -3 x = -4

8
Green Rose Center for Academe Inc.
Purok Gemelina Estaca Compostela Cebu
Mobile No. 0933-1617936 / Landline No. (032) 425-6216
E-mail add: greenroseacademe_119@yahoo.com, Website: http://grca.school
Government Recognition No:
PRE-ELEM: 04 S. 2017, ELEMENTARY: 5, S. 2017, JHS: 06, S.2017, SHS: 059 S.2018
School ID: 408281
MATHEMATICS 9- DRILL 3
(Topic: Solving Roots of Quadratic Equations)

Name: _____________________________Gr. & Sec. _________Date: ________


Parent’s Signature: ________________ Score: ________________
A. Directions: Find the sum and the product of the roots of the following
quadratic equations. Write your answer on the box being provided.

EQUATION SUM PRODUCT

𝑚2 + 2𝑚 + 3=0 1. 6.

5𝑚2 + 𝑚 + 7=0 2. 7.

𝑚2 − 8𝑚 + 16=0 3. 8.

𝑚2 − 𝑚 − 2=0 4. 9.

2𝑚2 − 9𝑚=0 5. 10.

B. Directions: Solve the unknown problem. Write your solutions in a clean


sheet of paper.

1. Construct a quadratic equation whose two roots are 3 and -1.


2. Construct a quadratic equation whose two roots are 2 and 3.
3. Construct a quadratic equation whose two roots are -4 and -1.
4. Transform the equation (𝑥 + 3)2 − 5𝑥 = 2 into quadratic equation.
5. Transform the equation (𝑥 − 1)2 − 8𝑥 = −2 into quadratic equation.

9
Green Rose Center for Academe Inc.
Purok Gemelina Estaca Compostela Cebu
Mobile No. 0933-1617936 / Landline No. (032) 425-6216
E-mail add: greenroseacademe_119@yahoo.com, Website: http://grca.school
Government Recognition No:
PRE-ELEM: 04 S. 2017, ELEMENTARY: 5, S. 2017, JHS: 06, S.2017, SHS: 059 S.2018
School ID: 408281
MATHEMATICS 9- ASSIGNMENT 3
(Topic: Solving Roots of Quadratic Equations)

Name: __________________________Gr. & Sec. ____________Date: ________


Parent’s Signature: ________________ Score: ________________

A. Directions: Solve the following equations using the sum and product of
roots. Write your solution and answer on the space being provided.

1. 7𝑥 2 -9x=8 2. 2𝑥 2 +3x-1=8𝑥+6

10
MATHEMATICS 9- LESSON 4
TOPIC: ILLUSTRATING QUADRATIC FUNCTIONS

Quadratic Functions - a function of the form f(x) = ax 2 + bx + c where


a, b and c are real numbers and a not equal to zero.

 The term ax2 in a quadratic function is called the quadratic term.


 The term bx is the linear term.
 The term c is the constant term.

The following examples illustrate quadratic functions:


1. f(x) = -2x 2 + x - 1
2. f(x) = x 2 + 3x + 2
3. f(x) = x(x-3) +5 (using distributive property of multiplication becomes
f(x) = x 2 -3x + 5)
4. f(x) = 2x 2
Determining each term, you will get:

Given General Quadratic Linear Constant


Form Term Term term
f(x) = x(x-3)+5 f(x) = x 2 - x2 -3x 5
3x+5
f(x) = x 2+x(x+2)- f(x) = 2x 2- 2x 2 -x 6
3(x-2) x+6

Values of a, b, and c in a Quadratic Function


Example 1: Determine the values of a, b, and c of the function x2+2x-3.
a=1
b=2
c = -3

11
Example 2: Determine the values of a, b, and c of the function h(x) = (x+1)
(x-4).

Step 1: Change the expression h(x) = (x+1) (x-4)


to the general form of quadratic = x(x)+x (-4) +1(x)+1(-4)
function using FOIL Method. = x2 -4x +x - 4
= x2 -3x – 4

Step 2: Determine the values of h(x) = x2 -3x – 4


a, b, and c. a=1
b = -3
c = -4

Thus, the values of a, b, and c of the given function are 1, -3, & -4
respectively.

Changing to General Form of the Quadratic Function


The general form of a quadratic equation is
y = 𝑎𝑥 2 + 𝑏𝑥 + 𝑐 where a, b and c are real numbers and a is not equal to
zero.
For example: y = 𝑥 2 + 5x – 30

Example 1: Change g(x) = (𝑥 − 2)2 + 1 into general form of the


quadratic functions

Step 1: Evaluate the exponent. g(x) = (𝑥 − 2)2 + 1


= (𝑥 2 -4x+4)+1

Step 2: Eliminate directly the g(x) = (𝑥 2 -4x+4)+1


parenthesis. = 𝑥 2 -4x+4+1

Step 3: Perform the operations to all g(x) = 𝑥 2 -4x+5


the like terms.

Thus, the general form of the given above is g(x) = 𝒙𝟐 -4x+5.

12
Green Rose Center for Academe Inc.
Purok Gemelina Estaca Compostela Cebu
Mobile No. 0933-1617936 / Landline No. (032) 425-6216
E-mail add: greenroseacademe_119@yahoo.com, Website: http://grca.school
Government Recognition No:
PRE-ELEM: 04 S. 2017, ELEMENTARY: 5, S. 2017, JHS: 06, S.2017, SHS: 059 S.2018
School ID: 408281
MATHEMATICS 9- DRILL 4
(Topic: Illustrating Quadratic Functions)
Name: _____________________________Gr. & Sec. _________Date: ________
Parent’s Signature: ________________ Score: _________________

A. Direction: Complete the table.

Given General Form Quadratic Linear Constant


Term Term Term

𝒇(𝒙) = 𝒙𝟐 + 𝟑𝒙 + 𝟏
1. 2. 3. 𝟏
𝟐
𝒈 (𝒙 ) = 𝒙 + 𝟗
4. 5. 6. 7.
𝟐
𝒉(𝒙) = 𝟗𝒙 − 𝟕𝒙 + 𝟒
8. 𝟗𝒙𝟐 9. 10.

B. Directions: Identify the following illustrates a quadratic function. Put (√)


on the number if it is quadratic and (×) if it is not.
1. 𝒇(𝒙) = 𝟑𝒙 + 𝟏

2. 𝒇(𝒙) = 𝒙𝟐 − 𝒙 + 𝟏

3. 𝒇(𝒙) = 𝟒𝒙𝟐 + 𝟒

4.𝒇(𝒙) = 𝟕𝒙𝟐 − 𝟑𝒙 + 𝟏𝟏

5.𝒇(𝒙) = 𝟑𝒙𝟑 + 𝟑𝒙 + 𝟎

13
MATHEMATICS 9- LESSON 5
TOPIC: REWRITING QUADRATIC FUNCTION

Finding the third term of quadratic function to make a perfect


square trinomial
Example 1: Find the third term of the function f(x) = x2 + 8x + _____?

Given f(x) = x2 + 8x + _____?

Step 1: Divide the 2nd term (coefficient of x f(x) = x2 + 8x + 42


which is 8) by 2 and square.

Step 2: Simplify the exponent of the third f(x) = x2 + 8x + 16


term.

Thus, the third term of the given function is 16.

Example 2: Find the third term of the function f(x) = x2 + 4x + ______?

Given f(x) = x2 + 4x + _____?

Step 1: Divide the 2nd term (coefficient f(x) = x2 + 4x + 22


of x which is 8) by 2 and square.

Step 2: Simplify the exponent of the f(x) = x2 + 4x + 4


third term.

Thus, the third term of the given function is 4.

Transform Quadratic Function from general form into standard form


The general form of a quadratic function f(x) = ax2 + bx + c may be
rewritten to its equivalent standard form f(x) = a(x-h)2 + k.
General Form: f(x) = ax2 + bx + c

14
Standard Form: f(x) = a(x-h)2 + k
If the quadratic function is in standard form, you can easily determine
the vertex (h, k), the line of symmetry and the opening of the parabola.
h - is the value of x.
k - is the value of y.
Example: Change f(x) = x2 + 10x + 9 to standard form.
Solution:

Given f(x) = x2 + 10x + 9


Step 1: Get the common factor of the f(x) = 1(x2 + 10x) + 9
first and second term.

Step 2: Inside the parenthesis, f(x) = 1(x2 + 10x) + 9


complete the square by dividing the f(x) = 1(x2 + 10x + 52) + 9 - 52
second term (coefficient of x which is
10) by 2, add it as your third term then
square and subtract the third terms
also outside the parenthesis.

Step 3: Simplify the exponents except f(x) = 1(x2 + 10x + 52) + 9 - 52


the x2. f(x) = 1(x2 + 10x + 25) + 9 – 25
Step 4: Inside the parenthesis, factor f(x) = 1(x2 + 10x + 25) + 9 – 25
the expression. f(x) = 1(x+5)(x+5) – 16

Step 5: Combine the factors. f(x) = 1(x+5) (x+5) – 16


f(x) = 1(x+5)2 -16
Thus, the standard form of the given expression is f(x) = 1(x+5)2 -16 and the
vertex is (-5, -16)

Transform Quadratic Function from standard form into general form


General Form: f(x) = ax2 + bx + c
Standard Form: f(x) = a(x-h)2 + k

15
Green Rose Center for Academe Inc.
Purok Gemelina Estaca Compostela Cebu
Mobile No. 0933-1617936 / Landline No. (032) 425-6216
E-mail add: greenroseacademe_119@yahoo.com, Website: http://grca.school
Government Recognition No:
PRE-ELEM: 04 S. 2017, ELEMENTARY: 5, S. 2017, JHS: 06, S.2017, SHS: 059 S.2018
School ID: 408281
MATHEMATICS 9- DRILL 5
(Topic: Rewriting Quadratic Function)
Name: _____________________________Gr. & Sec. _________Date: ________
Parent’s Signature: ________________ Score: ________________
A. Direction: Complete each quadratic function to make 𝑓(𝑥) a perfect square
trinomial.
1.𝑓 (𝑥 ) = 𝑥 2 − 8𝑥 + ________________
2.𝑓 (𝑥 ) = 𝑥 2 − 7𝑥 + ________________
3.𝑓 (𝑥 ) = 𝑥 2 + 9𝑥 + ________________
1
4.𝑓 (𝑥 ) = 𝑥 2 + 3 𝑥 + ________________
5
5.𝑓 (𝑥 ) = 𝑥 2 − 2 𝑥 + ________________

B. Direction: Simplify then rewrite each of the following into the form
𝑓(𝑥 ) = 𝑎𝑥 2 + 𝑏𝑥 + 𝑐.

2
1. 𝑓 (𝑥 ) = (𝑥 − 3)2 + 3
2. 𝑓 (𝑥 ) = (5𝑥 + 1)(𝑥 + 2) − (𝑥 − 3)(𝑥 + 2) + 5
3. 𝑓 (𝑥 ) = (𝑥 + 1)(𝑥 + 2) + 7
4. 𝑓 (𝑥 ) = (𝑥 − 8)(𝑥 + 9)
3
5. 𝑓 (𝑥 ) = ( + 𝑥)2 − 1
4

C. Direction: Rewrite each of the following quadratic function into standard


form.

1. 𝑓 (𝑥 ) = 𝑥 2 − 4𝑥 − 5
2. 𝑓 (𝑥 ) = 𝑥 2 + 9𝑥 − 4
3. 𝑓 (𝑥 ) = 3𝑥 2 − 10𝑥 + 1
1
4. 𝑓 (𝑥 ) = 2 𝑥 2 − 4𝑥 + 2
2
5. 𝑓 (𝑥 ) = −𝑥 2 − 𝑥 +
3

16
Green Rose Center for Academe Inc.
Purok Gemelina Estaca Compostela Cebu
Mobile No. 0933-1617936 / Landline No. (032) 425-6216
E-mail add: greenroseacademe_119@yahoo.com, Website: http://grca.school
Government Recognition No:
PRE-ELEM: 04 S. 2017, ELEMENTARY: 5, S. 2017, JHS: 06, S.2017, SHS: 059 S.2018
School ID: 408281
MATHEMATICS 9- ASSIGNMENT 5
(Topic: Rewriting Quadratic Function)

Name: __________________________Gr. & Sec. ____________Date: ________


Parent’s Signature: ________________ Score: ________________
A. Directions: Simplify each of the following quadratic expressions then write
in standard form 𝑓 (𝑥 ) = 𝑎(𝑥 − ℎ)2 + 𝑘. Write your solution and answer on
the space being provided.

1. 𝑓 (𝑥 ) = 6 − 3𝑥 − 8𝑥 2 + 5(2𝑥 2 − 𝑥 − 4)

2. 𝑓 (𝑥 ) = (3𝑥 − 2)2 − (2𝑥 − 3)2 − 10𝑥

17
MATHEMATICS 9- LESSON 6
TOPIC: GRAPHING OF QUADRATIC FUNCTION
Using Table of Values to represent Quadratic Function
A table and a graph can both be used to show solutions to a quadratic equation.
The graph and table below show points for the quadratic function.

𝒚 = 𝒙𝟐 − 𝒙 − 𝟔

Both representations of a quadratic equation can be used to find the solution.


The solutions to quadratic equations are called roots. Roots are the x-intercepts
(zeros) of a quadratic function.
For every quadratic equation, there is a related quadratic function. For example,
if you are given the quadratic equation x2 + 5x + 4 = 0, the related quadratic
function is f(x) = x2 + 5x + 4.
A quadratic equation may have two solutions, one solution, or no solution.
A table of values can be generated from a quadratic function by substituting the
x-values and calculating the values for f(x). When looking at a table of values for
a quadratic function, the x-intercepts represent the x-values where y = 0. This
corresponds to the x-values where f(x) is 0 in function notation.
How to use the table of values to get the y-values and complete the
table?
Follow this steps:
Example: Graph the quadratic function f(x) = x 2.
Step 1: Get the y-values using the table of values if x-values are -2, -1, 0, 1,
and 2.

X -2 -1 0 1 2
Y

18
Step 2: Solve the function by substituting the x-values to the function given.
f(x) = x2 f(x) = x2 f(x) = x2 f(x) = x2 f(x) = x2
= (-2)2 = (-1)2 = (0)2 = (1)2 = (2)2
=4 =1 =0 =1 =4
Step 3: Write the y-values in the table of values and write the coordinates.

X -2 -1 0 1 2
Y 4 1 0 1 4

Coordinates/Ordered Pair: (-2, 4), (-1, 1), (0, 0), (1, 1), and (2, 4)
Step 4: Graph the coordinates.

Finding Quadratic Function using the Table of Values


In finding the quadratic function using the table of values, follow this steps:
Example: Find the quadratic function of the given table of values below.
X -5 -4 -3 -2 -1 0 1
Y 6 2 0 0 2 6 12
Step 1: Get the first difference of y-values.
=2-6 =0-2 =0-0 =2-0 =6-2 = 12- 6
= -4 = -2 =0 =2 =4 =6
Step 2: Get the second difference of y-values.
= -2 -(-4) = 0 -(-2) =2-0 =4-2 =6-4
=2 =2 =2 =2 =2

19
Step 3: Equate 2a, (the second difference in f(x) = ax2 + bx + c) with the
second difference in y-values to find a.
2a = 2
2a/2 = 2/2 (divide 2 to both sides to eliminate the coefficient of a)
a=1
Step 4: Equate a + b with the difference of y-values of f(1) and f(0) to find the
b.

X -5 -4 -3 -2 -1 0 1
Y 6 2 0 0 2 6 12
=12 - 6 a+b=6
=6 1+b=6
b = 6 - 1 (transfer 1 to right side and positive 1
will become negative 1)
b=5
Step 5: The value of f(0) is c.

X -5 -4 -3 -2 -1 0 1
Y 6 2 0 0 2 6 12
f(0) = 6
so, c = 6
Step 6: Substitute the value of a, b, and c to the general form f(x) = ax 2 + bx +
c
f(x) = (1) x2 + (5) b + (6)

∴, the quadratic function of the given table of values is 𝒇(𝒙) = 𝒙𝟐 = 𝟓𝒙 + 𝟔.


LIST OF LINKS
Title: Graph and Table of Quadratic Function
Link:https://d1yqpar94jqbqm.cloudfront.net/styles/media_middle/s3/images/a1m6l6rimage1.jpg?itok=uu2d
NEuI

Title: Graphing the coordinates.

Link: http://cimg2.ck12.org/datastreams/f-
d%3A3315d4a481f4d445dd4b48c960100464fea5d3230167cb6966034ce9%2BIMAGE%2BIMAGE.1

20
Green Rose Center for Academe Inc.
Purok Gemelina Estaca Compostela Cebu
Mobile No. 0933-1617936 / Landline No. (032) 425-6216
E-mail add: greenroseacademe_119@yahoo.com, Website: http://grca.school
Government Recognition No:
PRE-ELEM: 04 S. 2017, ELEMENTARY: 5, S. 2017, JHS: 06, S.2017, SHS: 059 S.2018
School ID: 408281
MATHEMATICS 9- DRILL 6
(Topic: Graphing of Quadratic Function)
Name: _____________________________Gr. & Sec. _________Date: ________
Parent’s Signature: ________________ Score: _________________
A. Directions: Match the given x-values of the quadratic function f(x) = 4x2
from column A to their corresponding y-values from column B. Write your answer
on the space provided.
A B
_______1. 3 a. 16
_______2. -5 b. 400
_______3. 2 c. 36
_______4. -6 d. 144
_______5. -10 e. 100
B. Directions: Match the x and y-values from column A to their
corresponding quadratic function from column B. Write your answer on the space
provided.
A B
____1. x = -2, -1, 0, 1, 2 and y = 6, 1, -2, -3, -2 a. f(x) = x2 - 1
____2. x = 0, 1, 2, 3, 4 and y = 4, -5, -12, -17, -20 b. f(x) = x2 - 2x - 2
____3. x = -1, 0, 1, 2, 3 and y = 5, -1, 1, 11, 29 c. f(x) = 4x2 - 2x - 1
____4. x = -3, -2, -1, 0, 1 and y = 118, 61, 22, 1, -2 d. f(x) = 9x2 - 12x + 1
____5. x = -2, -1, 0, 1, 2, 3 and y = 5, 2, 1, 2, 5, 10 e. f(x) = x2 - 10x + 4
C. Direction: Graph the quadratic function 𝑓 (𝑥 ) = 𝑥 2 − 1 using graphing
paper and show your table of values.
(Note: 4 points for table of values and 6 points for the graph)

21
MATHEMATICS 9- LESSON 7
TOPIC: GRAPHING QUADRATIC FUNCTION II

Graph of Quadratic Function


The graph of a quadratic function is a U-shaped curve called a parabola. This
shape is shown below.
In graphs of quadratic functions, the
sign on the coefficient a affects
whether the graph opens up or
down. If a<0, the graph makes a
frown (opens down) and if a>0 then
the graph makes a smile (opens up).
This is shown beside.

Direction of Parabolas: The sign


on the coefficient a determines the
direction of the parabola.

Steps in sketching the graph of quadratic function in standard form:


To graph f(x) = a(x-h)2 + k:
Example: Sketch the graph of f(x) = x2 - 6x + 5.
Step 1: Determine the opening of the parabola by changing the general form of
the quadratic function into standard form, f(x) = a(x - h)2 + k. If a>0 the
parabola opens upward, if a<0, the parabola opens downward.
f(x) = x2 - 6x + 5
= 1(x2 - 6x) + 5
= 1(x2 - 6x + 32) + 5 - 32
= 1(x2 - 6x + 9) + 5 - 9
= 1(x - 3)(x - 3) - 4
= 1(x - 3)2 - 4
So a = 1 (a>0) Direction of the parabola: Opens Upward

22
Step 2: Determine the vertex of the parabola. The vertex is at (h, k)
Standard Form: f(x) = a(x - h)2 + k
The Standard form of the given function is f(x) = 1(x - 3)2 - 4
To find h: h = x
x-3=0
x=3
To find k: k = y
y = -4
Vertex: (3, -4)
Step 3: Determine the axis of symmetry. (x-value in vertex)
Axis of Symmetry: x = 3
Step 4: Find the x - intercept. Assign y = 0 then solve for x. (Use the general
form of the given quadratic function)
General form: f(x) = x2 - 6x + 5 or y = x2 - 6x + 5
y = x2 - 6x + 5
0 = x2 - 6x + 5 (Factor the trinomial)
0 = (x - 5)(x - 1) (Equate the factors into zero)
x-5=0 x-1=0 (Transfer -5 and -1 to the right
side and become positive)
x=5 x=1
x - intercepts: (5, 0) and (1, 0)
Step 5: Find the y - intercept. Assign x = 0 then solve for y. (Use the general
form of the given quadratic function)
General form: f(x) = x2 - 6x + 5 or y = x2 - 6x + 5
y = x2 - 6x + 5
y = 02 - 6(0) + 5
y = 5, y=intercept: (0, 5)

23
Step 6: Make a table of values for x and y and connect the points. (Use the
general form of the given quadratic function)
General form: f(x) = x2 - 6x + 5 or y = x2 - 6x + 5

X 0 1 2 3 4 5 6
Y
y = x2 - 6x + 5 y = x2 - 6x + 5 y = x2 - 6x + 5 y = x2 - 6x + 5
= (0)2 - 6(0)+ 5 = (1)2 - 6(1)+ 5 = (2)2 - 6(2)+ 5 = (3)2 - 6(3) +5
=0-0+5 =1-6+5 = 4 - 12 + 5 = 9 - 18 + 5
=0+5 = -5 + 5 = -8 + 5 = -9 + 5
=5 =0 = -3 = -4

y = x2 - 6x + 5 y = x2 - 6x + 5 y = x2 - 6x + 5
= (4)2 - 6(4) + 5 = (5)2 - 6(5) + 5 = (6)2 - 6(6) + 5
= 16 - 24 + 5 = 25 - 30 + 5 = 36 - 36 + 5
= -8 + 5 = -5 + 5 =0+5
= -3 =0 =5

The y-values are:


X 0 1 2 3 4 5 6
Y 5 0 -3 -4 -3 0 5

24
Step 7: Plot the vertex, the intercepts, and some points or coordinates and
connect these points/coordinates with a smooth curve.

LIST OF LINKS
Title: Graph of Quadratic Function
Link: https://textimgs.s3.amazonaws.com/boundless-
algebra/trygx7sythwhphxv8rg3.jpe#fixme
Title: Graph opens upward and downward
Link: https://s3-us-west-2.amazonaws.com/courses-images/wp-
content/uploads/sites/1861/2017/06/23162027/kvmcivr1sgqzuzdh4ofo.png
Title: Graph of Quadratic Function Figure 2
Link: https://1.bp.blogspot.com/-k4rMJ2UDTOk/XR2IArtH6cI/AAAAAAAABsA/w-
H1vMOAMxY6aTIx18Lszr_OYyIszwEEgCEwYBhgL/s1600/Graphing%2BQuadratic
%2BFunctions%2B%25282%2529.jpg

25
Green Rose Center for Academe Inc.
Purok Gemelina Estaca Compostela Cebu
Mobile No. 0933-1617936 / Landline No. (032) 425-6216
E-mail add: greenroseacademe_119@yahoo.com, Website: http://grca.school
Government Recognition No:
PRE-ELEM: 04 S. 2017, ELEMENTARY: 5, S. 2017, JHS: 06, S.2017, SHS: 059 S.2018
School ID: 408281
MATHEMATICS 9- DRILL 7
(Topic: Graphing Quadratic Function II)
Name: _____________________________Gr. & Sec. _________Date: ________
Parent’s Signature: ________________ Score: ________________
A. Directions: Identify the direction of the parabola (open upward or open
downward). Write your answer on the space being provided.
_______________________________1. 𝑓 (𝑥 ) = 𝑥 2 − 4𝑥 + 2
_______________________________2. 𝑓 (𝑥 ) = −𝑥 2 + 4𝑥 − 1
_______________________________3. 𝑓 (𝑥 ) = 𝑥 2 + 9𝑥 + 1
_______________________________4. 𝑓 (𝑥 ) = 2𝑥 + 𝑥 2 − 1
_______________________________5. 𝑓 (𝑥 ) = 𝑥 2 − 8𝑥 − 9
B. Directions: Find the axis of symmetry. Write your answer on the space
being provided.
_______________________________1. 𝑓 (𝑥 ) = 𝑥 2 − 4𝑥 + 2
_______________________________2. 𝑓 (𝑥 ) = −𝑥 2 + 4𝑥 − 1
_______________________________3. 𝑓 (𝑥 ) = 𝑥 2 + 9𝑥 + 1
_______________________________4. 𝑓 (𝑥 ) = 2𝑥 + 𝑥 2 − 1
_______________________________5. 𝑓 (𝑥 ) = 𝑥 2 − 8𝑥 − 9

C. Directions: Find the vertex of quadratic function. Write your answer on


the space being provided.
_______________________________1. 𝑓 (𝑥 ) = 𝑥 2 − 4𝑥 + 2
_______________________________2. 𝑓 (𝑥 ) = −𝑥 2 + 4𝑥 − 1
_______________________________3. 𝑓 (𝑥 ) = 𝑥 2 + 9𝑥 + 1
_______________________________4. 𝑓 (𝑥 ) = 𝑥 + 𝑥 2 − 1
_______________________________5. 𝑓 (𝑥 ) = 𝑥 2 − 8𝑥 − 9

26
Green Rose Center for Academe Inc.
Purok Gemelina Estaca Compostela Cebu
Mobile No. 0933-1617936 / Landline No. (032) 425-6216
E-mail add: greenroseacademe_119@yahoo.com, Website: http://grca.school
Government Recognition No:
PRE-ELEM: 04 S. 2017, ELEMENTARY: 5, S. 2017, JHS: 06, S.2017, SHS: 059 S.2018
School ID: 408281
MATHEMATICS 9- ASSIGNMENT 7
(Topic: Graphing Quadratic Function II)

Name: __________________________Gr. & Sec. ____________Date: ________


Parent’s Signature: ________________ Score: ________________
A. Directions: Sketch the graph of f(x) = x2 - 6x + 5. Write your solution and
answer on the space being provided.
(Note: Follow the steps and use graphing paper for the graph)

27
MATHEMATICS 9- LESSON 8
TOPIC: ILLUSTRATING DIRECT VARATION

Direct Variation also called as direct proportion is a relationship between two


variables x and y that can be written as y = kx, k ≠ 0. This situation occurs when
the ratio of two variables is constant. If y varies directly as x, this relation is
written as y ∝ x and read as y varies as x. The sign “ ∝ ” is read “varies as” and
is called the sign of variation.
Formula: y = kx, where k = constant of variation or the constant of
proportionality
Here, are an example in finding the constant of variation:
Example 1: y varies directly as x, find the constant of variation if y=30 when
x=6.
Step 1: Substitute the given x and y values, and solve for k.
y = kx
30 = k(6)
30 = 6k -------->>> divide both sides with 6 to get the value of k
30/6 = 6k/6
5 = k or k = 5 (constant of variation)

Example 2: The cost of a taxi fare (C) varies directly as the distance (D)
travelled. When the distance is 60 km, the cost is $35. Find the cost if the
distance is 96 km.
Solution: Because, C varies directly as D, we must have constant k such that:
C = kD
Step 1: Substitute C = 35 and D = 60 into the equation.
35 = k(60)
35 = 60k -------->>> divide both sides with 60 to get the value of k
35/60 = 60k/60
7/12 = k or k = 7/12 (constant of variation)

28
7
Step 2: The equation is C = D . Now substitute D=96 and find C.
12
7
C = kD ----->>> C = D
12
7
C= (96)
12
672
C=
12

C = 56 cost when the distance is 96 km.

Example 3: A recipe for 6 cupcakes needs 1 cup of flour. The number of


cupcakes (C) you can make varies directly with the amount of flour (F). How
many cupcakes can you make with 4 cups of flour?
Solution: Because, C varies directly as F, we must have constant k such that:
C = kF
Step 1: Substitute C = 6 and F = 1 into the equation.
6 = k(1)
6 = 1k or k = 6 (constant of variation)
Step 2: The equation is C = 6F. Now substitute F=4 and find C.
C = kF
C = 6F
C = 6(4)
C = 24 cupcakes you can make with 4 cups of flour.

29
Green Rose Center for Academe Inc.
Purok Gemelina Estaca Compostela Cebu
Mobile No. 0933-1617936 / Landline No. (032) 425-6216
E-mail add: greenroseacademe_119@yahoo.com, Website: http://grca.school
Government Recognition No:
PRE-ELEM: 04 S. 2017, ELEMENTARY: 5, S. 2017, JHS: 06, S.2017, SHS: 059 S.2018
School ID: 408281
MATHEMATICS 9- DRILL 8
(Topic: Illustrating Direct Variation)
Name: _____________________________Gr. & Sec. _________Date: ________
Parent’s Signature: ________________ Score: ________________
A. Directions: Write the following in equation from using 𝑘 as the constant
of variation. Write your answer on the space being provided.
_______________________________1. V varies directly with r
_______________________________2. A varies directly with s
_______________________________3. S varies directly with d
_______________________________4. R varies directly with h
_______________________________5. R varies directly with y
_______________________________6. B varies directly with s.
_______________________________7. P varies directly with x.
B. Directions: Solve the unknown problem. Write your answer on the space
being provided.
“Suppose Z directly proportional to r. If r=2 then Z=8”.
_______________________________8. Find the constant of proportionality.
_______________________________9. Find the equation of the variation.
_______________________________10. Find Z when t=9.
C. Directions: Find the constant variation. Write your answer on the space
being provided.
________________________1. y varies directly as x, and y = 9 when x = 54.
________________________2. y varies directly as x, and y = 6 when x = 54.
________________________3. y varies directly as x, and y = 8 when x = 64.
________________________4. y varies directly as x, and y = 12 when x = 144.
________________________5. y varies directly as x, and y = 1.5 when x = 550.

30
MATHEMATICS 9- LESSON 9
TOPIC: ILLUSTRATING DIRECT SQUARE VARATION
If x and y have a direct square relationship it means that the term x is
proportional to a constant multiplied by y squared. In such cases, the non-linear
relation between two quantities can be expressed by an equation y = kx2,
where y varies directly as the squared of x and the constant of variation is k.
This relation is called as direct square variation. Formula: y = kx2
SOLVING THE CONSTANT OF VARIATION
Example 1: Assume that y varies directly as the square of x. When x = 5, y =
40. Find the constant of variation.
Step 1: Substitute y = 40 and x = 5 to the formula.
y = kx2
40 = k(5)2
40 = k(25) -------->>> divide both sides with 25 to get the value of k
40/25 = 25k/25
40/25 = k -------->>> reduce to lowest term
8/5 = k or k = 8/5 (constant of variation)
SOLVING THE DIRECT SQUARE VARIATION
Example 2: The cost of making a circular dartboard (C) varies directly as the
square of its radius (r) in cm. A dartboard with radius 10 cm costs 1500 pesos.
Find the cost of the dartboard with radius equal to 42 cm.
Solution:
Since C varies directly as the square of r, then C = kr2
Step 1: Solve the constant variation. Substitute r = 10 and C = 1500.
C = kr2
1500 = k(10)2
1500 = k(100)
1500 = 100k -------->>> divide both sides with 100 to get the value of k
1500/100 = 100k/100
15 = k or k = 15 (constant of variation)

31
Step 2: Solve for C when r = 42, substitute k = 15
C = kr2
C = 15(42)2
C = 15(1764)
C = 26,460 pesos (cost of dartboard with 42 cm radius)
Example 3: If y varies directly as x2, and y = 24 when x = 4, how will you find
y when x = -4?
Solution:
Since y varies directly as the x2, then
y = kx2
Step 1: Solve the constant variation. Substitute x = 4 and y = 24.
y = kx2
24 = k(4)2
24 = k(16)
24 = 16k -------->>> divide both sides with 16 to get the value of k
24/16 = 16k/16
24/16 = k -------->>> reduce to lowest term
3/2 = k or k = 3/2
Step 2: Solve for y when x = -4, substitute k = 3/2
y = kx2
y = 3/2(-4)2
y = 3/2(16)
y = 48/2
y = 24

32
Green Rose Center for Academe Inc.
Purok Gemelina Estaca Compostela Cebu
Mobile No. 0933-1617936 / Landline No. (032) 425-6216
E-mail add: greenroseacademe_119@yahoo.com, Website: http://grca.school
Government Recognition No:
PRE-ELEM: 04 S. 2017, ELEMENTARY: 5, S. 2017, JHS: 06, S.2017, SHS: 059 S.2018
School ID: 408281
MATHEMATICS 9- DRILL 9
(Topic: Illustrating Direct Square Variation)
Name: _____________________________Gr. & Sec. _________Date: ________
Parent’s Signature: ________________ Score: ________________
A. Directions: Do as directed. Write your answer on the space being
provided.
Write the following in equation from using 𝑘 as the constant of variation.
_______________________________1. X varies directly with the square u
_______________________________2. A varies directly as the square of s
_______________________________3. S varies directly as the square of d
_______________________________4. R varies directly as the square of q
_______________________________5. H varies directly as the square of y
_______________________________6. B varies directly as the square of s
_______________________________7. K varies directly as the square x
Write an expression for each variation.
_______________________________8. The kinetic energy of a body in motion
varies directly with the square of its speed.
_______________________________9. A pizza varies directly as the square of its
radius.
_______________________________10. The distance required to stop a car
varies directly as the square of its speed.
B. Directions: Find the equation of variation in which y varies as the square
of x, given the following conditions. Write your answer on the space being
provided.
________________1. y=3 when x=6 _______________2. y=10 when x=10
________________3. y=3 when x=1.5 _______________4. y=0.5 when x=0.1
________________5. y=5.625 when x=1.5

33
Green Rose Center for Academe Inc.

Purok Gemelina Estaca Compostela Cebu


Mobile No. 0933-1617936 / Landline No. (032) 425-6216
E-mail add: greenroseacademe_119@yahoo.com, Website: http://grca.school
Government Recognition No:
PRE-ELEM: 04 S. 2017, ELEMENTARY: 5, S. 2017, JHS: 06, S.2017, SHS: 059 S.2018
School ID: 408281
MATHEMATICS 9- ASSIGNMENT 9
(Topic: Illustrating Direct Square Variation)

Name: _____________________________Gr. & Sec. _________Date: ________


Parent’s Signature: ________________ Score: _________________

A. Directions: Solve the problem. Write your solution and answer on the
space being provided.

1. If 𝑦 varies directly as the square of 𝑥, what is the effect on 𝑦 if 𝑥 is


decreased by 20%?

34
MATHEMATICS 9- LESSON 10
TOPIC: ILLUSTRATING INVERSE VARATION
Inverse Variation
The statement "y varies inversely as x means that when x increases, y decreases
by the same factor.
Key Ideas of Inverse Variation
1. We say that y varies inversely with x if y is expressed as the product of
some constant number k and the reciprocal of x.
2. However, the value of k can’t equal zero, i.e. k ≠ 0.
3. Isolating k on one side, it becomes clear that k is the fixed product of xx and
y. That means, multiplying x and y always yields a constant output of k.
Formula in finding the constant of variation:
k = xy
Example 1: Suppose that y varies inversely as x and that y = 8 when x = 3.
Find the constant of variation.
Solution:
Substitute y = 8 and x = 3 to the formula:
k = xy
= (3)(8)
= 24

Example 2: Suppose that y varies inversely as x and that y = 10 when x = 5/2.


Substitute y = 10 and x = 5/2 to the formula:
k = xy
= (5/2)(10)
= 50/2
= 25
Formula in Finding the Inverse Variation:
y = k/x

35
Example 3: y varies inversely as x. y = 4 when x = 2. Determine the inverse
variation equation. Then determine y when x = 16.
Solution:
Step 1: Substitute y = 4 and x = 2 to the formula to get the constant of
variation.
k = xy
= (2)(4)
k=8

Step 2: Substitute k = 8 and x = 16 to the formula to get the inverse variation.


y = k/x
y = 8/16
y = 1/2 or 0.5

Example 4: The time, t, required to empty a tank varies inversely as the rate, r,
of pumping. If a pump can empty a tank in 150 minutes at a rate of 400 gallons
per minute, how long will it take to empty a tank at 500 gallons per minute?
Solution:
Since t varies inversely as r, then
t = k/r
Step 1: Substitute t = 150 and r = 400 to the formula to get the constant of
variation.
k = tr
k = (150)(400)
k = 60000
Step 2: Substitute k = 60000 and r = 500 to the formula to get the inverse
variation.
t = 60000/500

36
Green Rose Center for Academe Inc.
Purok Gemelina Estaca Compostela Cebu
Mobile No. 0933-1617936 / Landline No. (032) 425-6216
E-mail add: greenroseacademe_119@yahoo.com, Website: http://grca.school
Government Recognition No:
PRE-ELEM: 04 S. 2017, ELEMENTARY: 5, S. 2017, JHS: 06, S.2017, SHS: 059 S.2018
School ID: 408281
MATHEMATICS 9- DRILL 10
(Topic: Illustrating Inverse Variation)
Name: _____________________________Gr. & Sec. _________Date: ________
Parent’s Signature: ________________ Score: ________________
A. Directions: Solve the unknown problem. Write your answer on the space
being provided.

Assume that y varies inversely as x. When x=7 and y=90.


_______________________________1. What is the equation of variation?
_______________________________2. What is the value of y when x=4?
Assume that u varies inversely as 3v+5. When v=7 and u=8
_______________________________4. Find an equation connecting u and v.
_______________________________5. Find the value of v when u=13.
_______________________________6. Suppose that y varies inversely as x and
that y = 9 when x = 3. Find the constant of variation.
_______________________________7. Suppose that y varies inversely as x and
that y = 8 when x = 7. Find the constant of variation.
________________________________8. Suppose that y varies inversely as x and
that k= 2 when x = 5. What is the equation of variation?
________________________________9. Suppose that y varies inversely as x and
that k= -7 when x = -2. What is the equation of variation?
________________________________10. Suppose that y varies inversely as x
and that k=-1 when x=-3. What is the equation of variation?

37
MATHEMATICS 9- LESSON 11
TOPIC: ILLUSTRATING JOINT VARIATION
Joint Variation – a mathematical sentence that describes the relationship
between two quantities that are jointly proportional; “y varies jointly as x and z”
is written as 𝑦 = 𝑘𝑥𝑧 where 𝑘 is the constant of variation.
Here, are an example in finding the constant of variation:
Example 1: y varies jointly as x and z, find the constant of variation if y=30
when x=5 and z=2.
Step 1: Substitute the given x, y and z values, and solve for k.
y = kxz
30 = k(5)(2)
30 = 10k -------->>> divide both sides with 10 to get the value of k
30/10 = 10k/10
3 = k or k = 3 (constant of variation)
Example 2: If 𝑧 varies jointly as 𝑥 and 𝑦 and 𝑧 = 48 when 𝑥 = 4 and 𝑦 = 15,
what is the equation of variation?
Step 1. Translate the variation statement to an equation. 𝑧 varies jointly as 𝑥 and
𝑦 𝑧 = 𝑘𝑥𝑦
Step 2. Solve for 𝑘 using the values of 𝑧, 𝑥, and 𝑦.
𝑧 = 48, 𝑥 = 4, 𝑦 = 15
z=kxy
48 = 𝑘(4)(15)
48 = 60𝑘/60 -------->>> divide both sides with 60 to get the value of k
48/ 60 = 𝑘
4 /5 = 𝑘
Step 3. Substitute the value of 𝑘 to the original equation to find the equation of
variation.
4
𝑧 = 𝑥𝑦
5
𝟒
Thus, the equation of variation is 𝒛 = 𝟓 𝒙𝒚.

38
Example 3: The volume (𝑉) of a prism with a constant width varies jointly
as the length (𝑙) and height (ℎ). If the volume is 1 080 𝑚3 when the length is 24
m and the height is 5 m, what is the volume if the length and height are each
increased by 3 m each?
Step 1. Translate the variation statement to an equation.
The volume (𝑉) of a prism with a constant width varies jointly as the length (𝑙)
and height (ℎ).
𝑉 = 𝑘𝑙ℎ
Step 2. Solve for 𝑘 using the values of 𝑉, 𝑙, and ℎ.
𝑉 = 1 080, 𝑙 = 24, ℎ = 5
𝑉 = klh
1 080 = 𝑘(24)(5)
1080 120𝑘
= -------->>> divide both sides with 120 to get the value of k
120 120

9=𝑘
Step 3: Substitute the value of 𝑘 to the original equation to determine the
equation of variation.
𝑉 = klh
V=9𝑙ℎ
Step 4: Solve the unknown. Note that the length and height are each increased
by 3.
𝑉 =?, 𝑙 = 24 + 3 = 27, ℎ = 5 + 3 = 8
𝑉 = 9(27) (8)
𝑉 = 1 944
Therefore, the volume of the prism is 1 944 𝑚3 when the length and height are
each increased by 3 m.

39
Green Rose Center for Academe Inc.
Purok Gemelina Estaca Compostela Cebu
Mobile No. 0933-1617936 / Landline No. (032) 425-6216
E-mail add: greenroseacademe_119@yahoo.com, Website: http://grca.school
Government Recognition No:
PRE-ELEM: 04 S. 2017, ELEMENTARY: 5, S. 2017, JHS: 06, S.2017, SHS: 059 S.2018
School ID: 408281
MATHEMATICS 9- DRILL 11
(Topic: Illustrating Joint Variation)
Name: _____________________________Gr. & Sec. _________Date: ________
Parent’s Signature: ________________ Score: ________________
A. Directions: Translate each statement into a formula. Use 𝑘 as the constant
of variation. (Note: 2 points for each number)
_____________________1. T varies jointly as s and p.
_____________________2. V varies jointly as l,w, and h
_____________________3. The total force, F, of a liquid varies jointly as the
surface area, A, the depth of liquid, h, and density, d.
_____________________4. The volume, V, of a cylinder varies jointly as its
height, h, and the square f its radius, r.
_____________________5. The heat, H, produced by an electric lamp varies
jointly as the resistance, R, and the square of the current, C.
B. Direction: Write TRUE if the statement is correct and FALSE if it is not.
____________1. If a varies directly as b, then a=kb.
____________2. If a is jointly proportional to b and c, then a=bc.

____________3. If a is directly proportional to the square root of c, then a=k√𝑐.


____________4. If b is directly proportional to a, then b=k𝑎2 .
𝑘𝑐
____________5. If a is jointly proportional to c and to the square of b, then a= 2.
𝑏

ASSIGNMENT 11
(Topic: Illustrating Joint Variation)
Name: _____________________________Gr. & Sec. _________Date: ________
Parent’s Signature: ________________ Score: ________________
A. Direction: Solve the problem.
1. The work (𝑤) done when lifting an object varies jointly as the mass (𝑚)
of the object and the height (ℎ) it is lifted. If 2 250 joules was done by
lifting a 75-kg object at a height of 2 m, how much work is done in lifting
a 120-kg object at a height of 0.8 m lower?

40
MATHEMATICS 9- LESSON 12
TOPIC: ILLUSTRATING COMBINED VARATION
Combined Variation – the mathematical representation of a combined
variation depends on the variation statement, e.g. “𝑦 varies directly as 𝑥 and
𝑘𝑥
inversely as 𝑧” is written as 𝑦 = 𝑧 .

Here are an example in finding the constant of variation:


Example 1: y varies directly as x and inversely as z, find the constant of
variation if y=100 when x=5 and z=2.
Step 1: Substitute the given x, y and z values, and solve for k .
𝑘𝑥
y=
𝑧
𝑘(5)
100 = 2

200 = 5k -------->>> divide both sides with 5 to get the value of k


200 5𝑘
=
5 5

40 = k or k = 40 (constant of variation)
Example 2: If 𝑦 varies directly as 𝑥 and inversely as 𝑧 and 𝑦 = 12 when 𝑥 = 3
and 𝑧 = 5, what is the equation of variation?
Step 1: Translate the variation statement to an equation.
𝑘𝑥
y varies directly as x and inversely as z 𝑦= 𝑧

Step 2: Solve for 𝑘 using the values of 𝑦, 𝑥, and 𝑧.


𝑘𝑥
𝑦 = 12, 𝑥 = 3, 𝑧 = 5 𝑦= 𝑧

𝑘(3)
12 = 5

60 = 3𝑘
20 = 𝑘
Step 3: Substitute the value of 𝑘 to the original equation to determine the
equation of variation.
20𝑥
𝑦=
𝑧
20𝑥
Thus, the equation of variation is 𝑦 = .
𝑧

41
Example 3: If 12 men can make 3 baskets in 5 hours, how long will it take 25
men to make 10 baskets?
Step 1: Analyze the problem and make an equation based on the type of
variations. The time (𝑡) it takes to make baskets varies directly as the number of
baskets (𝑏) to be made and inversely as the number of men (𝑚) working. This is
true since as the number of baskets increases or decreases, the time will
increase or decrease. On the other hand, as the number of men increases or
decreases, the time will decrease or increase. The problem is represented by the
𝑘𝑏
equation 𝑡 = 𝑚 .

Step 2: Solve for 𝑘 using the values of 𝑡, 𝑏, and 𝑚.


𝑡 = 5, 𝑏 = 3, 𝑚 = 12
𝑘𝑏
𝑡= 𝑚
𝑘(3)
5= 12

60 = 3𝑘
20 = 𝑘
Step 3: Substitute the value of 𝑘 to the original equation to determine the
equation of variation.
𝑘𝑏
𝑡= 𝑚
20𝑏
𝑡=
𝑚

Step 4: Solve the unknown.


𝑡 =? , 𝑏 = 10, 𝑚 = 25
20(10)
𝑡= 25
200
𝑡=
25

𝑡=8
Therefore, it will take 8 hours for 25 men to make 10 baskets.

42
Green Rose Center for Academe Inc.
Purok Gemelina Estaca Compostela Cebu
Mobile No. 0933-1617936 / Landline No. (032) 425-6216
E-mail add: greenroseacademe_119@yahoo.com, Website: http://grca.school
Government Recognition No:
PRE-ELEM: 04 S. 2017, ELEMENTARY: 5, S. 2017, JHS: 06, S.2017, SHS: 059 S.2018
School ID: 408281
MATHEMATICS 9- DRILL 12
(Topic: Illustrating Combined Variation)
Name: _____________________________Gr. & Sec. _________Date: ________
Parent’s Signature: ________________ Score: ________________
A. Directions: Translate each statement into a formula. Use 𝑘 as the constant
of variation. Write your answer on the space being provided.
_______________________________1. V varies directly as d and inversely as t.
_______________________________2. A varies directly as the square of b and
inversely as c.
_______________________________3. Q varies directly as m and inversely as
the square of t.
_______________________________4. The pressure, P, of gas varies directly as
its temperature, T, and inversely as the volume, V.
_______________________________5. The centrifugal force, F, of an object
moving in a circular path is directly proportional to the square of its velocity, v,
and inversely proportional to the diameter, d, of its path.
B. Directions: Write TRUE if the statement is correct and FALSE if it is not.
Write your answer on the space being provided.
__________1. If a is inversely proportional to b, then a=bk.
𝑘𝑏
__________2. If a varies directly as b and inversely as c, then a= 𝑐 .

__________3. If a varies directly as c and inversely as the square root of b, then


𝑘𝑐
a=𝑏2.

__________4. If b varies directly as a and inversely as the square of c, then


b=𝑘𝑎√𝑐.
__________5. Combined Variation is the mathematical representation of a
combined variation depends on the variation statement, e.g. “𝑦 varies directly as
𝑥 and inversely as 𝑧” is written as 𝑦 = 𝑘𝑥𝑧.

43
MATHEMATICS 9- LESSON 13
APPLYING LAWS OF EXPONENTS

LAWS OF EXPONENT
1. Product Rule of Exponents EXAMPLES:
If m and n are integers and a ≠ 0, 1. (x7)(x3) = x7+3 = x10
then
2. (74)(72) = 76 = 117,649
(am)(an) = am+n
3. (2x3)(10x2)(x) = (2)(10)(x3+2+1) =
20x6

2. Power of a Power Rule EXAMPLES:


If m and n are integers and a ≠ 0, 1. (x2)6 = x2(6) = x12
then
2. (23)3 = 23(3) = 29 = 512
(am)n = amn
3. (y3)2 = y3(2) = y6

3. Power of a Product
If n is an integer and a or b ≠ 0, then
(ab)n = (an)(bn)

EXAMPLES:
1. (3x)4 = (34)(x4) = (81)(x4) = 81x4
2. (2x2y3)3 = (23)(x2(3))(y3(3)) = (8)(x6)(y9) = 8x6y9
3. (3x4y3z2)3 = (33)(x4(3))(y3(3))(z2(3)) = (27)(x12)(y9)(z6) = 27x12y9z6

44
4. Quotient Rule
If m and n are integers and a ≠ 0, then
am/an = am-n

EXAMPLES:
1. x5/x3 = x5-3 = x2
2. 26/22 = 26-2 = 24 = 16
3. (76/73)(24/23) = (76-3)(24-3) = (73)(21) = (343)(2) = 686

5. Power of Quotient Rule


If n is an integer and a and b ≠ 0, then
(a/b)n = (an/bn)

EXAMPLES:
1. (x/5)3 = (x3/53) = x3/125
2. (3/4x3)2 = (32/42x3(2)) = 9/16x6
3. (-2x3/3y2)3 = (-23x3(3)/33y2(3)) = -8x9/27y6

45
Green Rose Center for Academe Inc.
Purok Gemelina Estaca Compostela Cebu
Mobile No. 0933-1617936 / Landline No. (032) 425-6216
E-mail add: greenroseacademe_119@yahoo.com, Website: http://grca.school
Government Recognition No:
PRE-ELEM: 04 S. 2017, ELEMENTARY: 5, S. 2017, JHS: 06, S.2017, SHS: 059 S.2018
School ID: 408281
MATHEMATICS 9- DRILL 13
(Topic: Applying Laws of Exponents)
Name: _____________________________Gr. & Sec. _________Date: ________
Parent’s Signature: ________________ Score: ________________
A. Directions: Simplify the following exponential expressions and write your
answer with positive exponents. Write your solution and answer on the
space being provided.
1. (2𝑥 3 )(5𝑥 7 ) 2. (8𝑥 3 )(5𝑥 7 𝑦)(𝑥 10 )

3. (3𝑥 2 𝑦𝑧 5 )(7𝑥 3 𝑦𝑧)(3𝑥𝑦𝑧 3 ) 4. (8𝑥 2 )3

5.(5𝑥𝑦𝑧 5 )2 6. (6𝑥 2 4𝑦 3 3𝑧 2 )5

3𝑥𝑦 8𝑥𝑦 3
7.( )2 8. ( )
6𝑥 2 5𝑦2 2𝑧 6 3𝑥𝑧𝑦

46
6𝑧𝑦 3 𝑥 2 9𝑎𝑏3
9. 10. (2𝑐𝑏2 )6
3𝑥𝑧𝑦 2

38 (63) 107 (24 )(32)


11. 12.
122 206

2
13. ( )3 14. (23 )3
7

15. ((𝑥 0 )1 )2

MATHEMATICS 9- ASSIGNMENT 13
(Topic: Applying Laws of Exponent)

Name: __________________________Gr. & Sec. ____________Date: ________


Parent’s Signature: ________________ Score: ________________
A. Directions: Analyze and solve the word problem. Write your solution and
answer on the space being provided.

1. The area of a rectangle is represented by the exponential expression


(64𝑥 2 𝑦 6 )−2 square units while its width is (8𝑥 −2 𝑦 2 )−1 units. What is the
expression that represents its length?

47
MATHEMATICS 9- LESSON 14
TOPIC: SIMPLIFYING ROOTS OF A NUMBER AND FRACTIONAL
EXPONENTS

Roots of a number
Recall that squaring a number means using that number two times as a factor.
Example:
92 = (9)(9) = 81
(-9)2 = (-9)(-9) = 81

The opposite in squaring a number is finding its square root. To find the square
root of 81, you must find two equal factors whose product is 81.
Example 1:

Simplify √144
Solution:

Since 122 = 144, therefore √144= 12

Example 2:

Simplify -√169
Solution:

Since 132 = 169, -(13), then -√169= -13

Example 3:

Simplify √225
Solution:

Since 152 = 225, therefore √225 = 15

48
Example 4:

Simplify √169𝑚2 𝑛2
Solution:

Since 132 = 169 and m2n2 = mn, therefore √169𝑚2 𝑛2 = 13mn

Example 5:

Simplify √16𝑚2
Solution:

Since 42 = 16 and m2 = m, therefore √16𝑚2 = 4m

The table below gives the squares and the cubes of first 12 counting
numbers:

N n2 n3
1 1 1
2 4 8
3 9 27
4 16 64
5 25 125
6 36 216
7 49 343
8 64 512
9 81 729
10 100 1000
11 121 1331
12 144 1728

FRACTIONAL EXPONENTS
To change fractional exponents to radicals, the denominator becomes the index
and the numerator is the exponent of the radicand.

49
Green Rose Center for Academe Inc.
Purok Gemelina Estaca Compostela Cebu
Mobile No. 0933-1617936 / Landline No. (032) 425-6216
E-mail add: greenroseacademe_119@yahoo.com, Website: http://grca.school
Government Recognition No:
PRE-ELEM: 04 S. 2017, ELEMENTARY: 5, S. 2017, JHS: 06, S.2017, SHS: 059 S.2018
School ID: 408281
MATHEMATICS 9- DRILL 14
(Topic: Simplifying Roots of a Number and Fractional Exponents)
Name: _____________________________Gr. & Sec. _________Date: ________
Parent’s Signature: ________________ Score: ________________
A. Directions: Solve and complete the table below. Write your answer on the
box being provided.

𝒏 𝒏𝟐 𝒏𝟑 𝒏𝟒

1. 5.
21 𝟏𝟗𝟒 𝟒𝟖𝟏

2. 8.
30x 𝟐𝟕 𝟎𝟎𝟎𝒙𝟐

3. 6. 9.
-4xy

4. 7. 10.
-9𝑥 2

𝒏 √𝒏

27m6 1.

25x 8 y 2 2.

(16a3 b6 c9 ) 3.

100m6 4.

225z6 5.

50
MATHEMATICS 9- LESSON 15
TOPIC: SIMPLIFYING LAWS OF RADICALS
A radical expression is an expression involving the root symbol (√). The
root symbol itself, is called the radical. The radicand, represented by the value
inside the root symbol is the number that will be operated on, and the index of
the root represented by the value outside.
To solve a radical expression, we can break the radicand into its prime
factors. If the radicand can be written as an exponent raised to a number equal
to the index, then the exponent will cancel out. If the radicand cannot be broken
down into a prime factor raised to an exponent equal in number to the index,
then the following Radical Rules can be applied.
Laws of Radicals
Assume that when n is even, a > 0.
1. ( √𝒂)𝒏 = 𝒂
𝒏

Example:
3
a. √−53 = -5
𝒏 𝒏
2. √𝒂𝒃 = √𝒂 × √𝒃
𝒏

Example:

3
a. 3√(−8𝑚3 )(27)= √−8𝑚3 × √27
3

= (-2m)(3)
= -6m
𝐧
𝐧 𝐚 √𝐚
3. √𝐛 = 𝐧
√𝐛

Examples:
𝟒 𝟏𝟔
a. √𝟖𝟏

𝟒
√𝟏𝟔
= 𝟒
√𝟖𝟏
𝟐
=
𝟑

51
16𝑚4
b. √ 25

√16𝑚2
=
√25
𝟒𝒎
= 𝟓
𝒎𝒏
4.
√ 𝒎𝒏
√𝒂 = √𝒂
Examples:
√3
a. √729

= √729
6

=3
√4
b. √256
8
= √256𝑚16
= 2𝒎𝟐

The simplified form of a radical expression would require;


1. NO prime factor of a radicand that has an exponent equal to or greater
than the index.
2. NO radicand contains a fraction.
3. NO denominator contains a radical sign.

Note:
𝒏
1. √𝒂 + 𝒃 ≠ √𝒂 + √𝒃
𝒏 𝒏

𝒏 𝒏
2. √𝒂 − 𝒃 ≠ √𝒂 − √𝒃
𝒏

3. √𝒂𝒏 + 𝒃𝒏 ≠ 𝒂 + 𝒃
𝒏

52
Green Rose Center for Academe Inc.
Purok Gemelina Estaca Compostela Cebu
Mobile No. 0933-1617936 / Landline No. (032) 425-6216
E-mail add: greenroseacademe_119@yahoo.com, Website: http://grca.school
Government Recognition No:
PRE-ELEM: 04 S. 2017, ELEMENTARY: 5, S. 2017, JHS: 06, S.2017, SHS: 059 S.2018
School ID: 408281
MATHEMATICS 9- DRILL 15
(Topic: Simplifying Laws of Radicals)
Name: _____________________________Gr. & Sec. _________Date: ________
Parent’s Signature: ________________ Score: ________________
A. Directions: Simplify the following radical expressions using the laws of
radicals. Write your solution and answer on the space being provided.
(Note: 1 point for Nos. 1-5 and 2 points for Nos. 6-10)
1. √121 2. √𝑥 6

3. √100𝑥 4 4. √3364

3
5. √729𝑚3 6. √49𝑎2 √𝑏4

3 3 8𝑥 2
7.√ √125𝑎12 8. √( 3𝑦 )2

2𝑥
√3 √4
9. √1331𝑚9 𝑛5 10. √( 33 )4
2𝑥

ASSIGNMENT 15
(Topic: Simplifying Laws of Radicals)

Name: __________________________Gr. & Sec. ____________Date: ________


Parent’s Signature: ________________ Score: ________________
A. Directions: Solve the word problem and express your answer as a radical
in simplest form. Write your solution and answer on the space being
provided.

1. What is the area of a square whose side is represented by the radical


expression ( 4√4𝑥 2 𝑦 3 ) units?

53
MATHEMATICS 9- LESSON 16
TOPIC: SIMPLIFYING RADICAL EXPRESSIONS

A radical expression is said to be simplest form whenever the following


conditions are satisfied;
1. Radicand is the smallest possible integer and the radicand must be an integer
that does not contain a perfect nth factor other than one.
2. The denominator does not contain any radicals.
3. The index of the radical has been reduced.

When simplifying a radical, the work will be easier if you can identify a factor
which is a perfect nth power of the radicand.

Example 1: Simplify √63.

√63 = √9 × 7

= √9 × √7

= 𝟑√𝟕

Example 2: Simplify 12 √88.


3

12 √88 = 12 √8 × 11
3 3

= 12 √8 × √11
3 3

=12(2) √11
3

= 𝟐𝟒 √𝟏𝟏
𝟑

54
Examples of Perfect Squares

Numbers that are perfect Algebraic expressions that are


squares: perfect squares:

4 x2

9 y4

16 m6

25 k12

36 a6b4c2

Example 3: Simplify √𝑚7 .

√𝑚7 = √𝑚6 × √𝑚

= 𝒎𝟑 √𝒎

Example 4: Simplify √80𝑎4 𝑏11 .

√80𝑎4 𝑏11=√16 × 5 × 𝑎4 × 𝑏11

= √16 × √5 × √𝑎4 × √𝑏10 × √𝑏

= 𝟒𝒂𝟐 𝒃𝟓 √𝟓𝒃

Example 5: Simplify 3√108𝑥 4 𝑦 8 𝑧 11 .

√108𝑥 4 𝑦 8 𝑧 11= √27 × 4 × 𝑥 3 × 𝑥 × 𝑦 6 × 𝑦 2 × 𝑧 9 × 𝑧 2


3 3

3 3 3
= √27 × √4 × √𝑥 3 × √𝑥 × 3√𝑦 6 × 3√𝑦 2 × √𝑧 9 × √𝑧 2
3 3 3

=𝟑𝒙𝒚𝟐 𝒛𝟑 √𝟒𝒙𝒚𝟐 𝒛𝟐
𝟑

55
Example 6:

Example 7:

Example 8:

LIST OF LINKS
Title: Applying the product and quotient rule for radicals

Link: https://saylordotorg.github.io/text_elementary-
algebra/section_11/9eaf448bc05dc22f2b03a2a65e91ea3d.jpg

Title: Applying the product rule for radicals

Link: http://saylordotorg.github.io/text_elementary-
algebra/section_11/db932aa0613e791d818f0b1e597aa631.jpg

Title: Simplest form of Radicals

Link: https://www.chilimath.com/wp-content/uploads/2017/06/41.png

56
Green Rose Center for Academe Inc.
Purok Gemelina Estaca Compostela Cebu
Mobile No. 0933-1617936 / Landline No. (032) 425-6216
E-mail add: greenroseacademe_119@yahoo.com, Website: http://grca.school
Government Recognition No:
PRE-ELEM: 04 S. 2017, ELEMENTARY: 5, S. 2017, JHS: 06, S.2017, SHS: 059 S.2018
School ID: 408281
MATHEMATICS 9- DRILL 16
(Topic: Simplifying Radical Expressions)
Name: _____________________________Gr. & Sec. _________Date: ________
Parent’s Signature: ________________ Score: ________________
A. Directions: Fill in the blank with <,> or =. Write your answer on the space
being provided.
1. √3𝑚_______√𝑚(3)
(81)(14𝑚)
2. 9√7𝑚______√
2
5𝑚 5
3. √ _______√7𝑚
7
3
√3
4. √3_______ √3
6

5. 3√45𝑥𝑦𝑧 3_________𝑧 3√45𝑥𝑦


B. Directions: Simplify each expression. Write your solution and answer on the
space being provided.
(Note: two points for every number)
5𝑎𝑏√125𝑑9 𝑒 7 3
1. 2. 4 𝑥 4 √128𝑦 3
10√8𝑎5 𝑏2

3𝑥 2 √108𝑦 7
3. 8𝑥 2 √18𝑥 6 𝑦 3 4.
12𝑦√18𝑥 5

15𝑚𝑛√8𝑚5 𝑝11
5. 3
10𝑚𝑝2 √54𝑛3

57
MATHEMATICS 9- LESSON 17
TOPIC: PERFORMING ADDITION AND SUBTRACTION OF
RADICAL EXPRESSIONS
Radicals expressions with the same indices and radicands are called similar
radicals. Such radicals are added or subtracted in the same way that monomials
are added or subtracted. This procedure is much like combining similar terms.
The steps in adding and subtracting Radical are:
Step 1. Simplify radicals. If you don't know how to simplify radicals go to
Simplifying Radical Expressions
Step 2. Combine like radicals.
Example 1:

Example 2:

Simplify 6√11 + 2√11 − 3√11


Solution:

6√11 + 2√11 − 3√11

= (6 + 2 − 3)√11

=(8 − 3)√11

= 𝟓√𝟏𝟏
Example 3:

Simplify 23 √10 − 14 √10 + √10


3 3 3

Solution: 23 √10 − 14 √10 + √10


3 3 3

= (23 − 14 + 1) √10
3

= (9 + 1) √10
3

= 𝟏𝟎 √𝟏𝟎
𝟑

58
Example 4: Simplify 3√2𝑥 + 4√2𝑥 − √2𝑥

Solution: 3√2𝑥 + 4√2𝑥 − √2𝑥

= (3 + 4 − 1)√2𝑥

= (7 − 1)√2𝑥

= 𝟔√𝟐𝒙

Example 5: Simplify 2𝑚√2 + 4𝑚√5 − 𝑚√3


Solution:

The expression 2𝑚√2 + 4𝑚√5 − 𝑚√3 cannot be simplified further because the
radicals are not similar, there are no common factors and each radicand are in
simplest form
If each radical in a radical expression is not in simplest form, you need to
simplify them first before you can find any like terms to combine.
Example 6: Simplify each pair of radical expressions to make them similar.

√45𝑥 and √20𝑥

√32𝑚2 and 2𝑚√8

4𝑎√12𝑏 and 5√75𝑎2 𝑏

Solutions: √45𝑥 and √20𝑥

= √9(5𝑥) and √4(5𝑥)

= 𝟑√𝟓𝒙 and 𝟐√𝟓𝒙 --> they are now similar because they are the same both
index and radicand

√32𝑚2 and 2𝑚√8

= √16𝑚2 (2) and 2𝑚√4(2)

=𝟒𝒎√𝟐 and 𝟒𝒎√𝟐 --> they are now similar because they are the same both
index and radicand

4𝑎√12𝑏 and 5√75𝑎2 𝑏

= 4𝑎√4(3𝑏) and 5√25𝑎2 (3𝑏)

= 𝟖𝒂√𝟑𝒃 and 𝟐𝟓𝒂√𝟑𝒃 --> they are now similar because they are the same
both index and radicand

59
Example 7:

Example 8:

Example 9:

60
Example 10:

LIST OF LINKS
Title: Combining like terms
Link:https://tse2.mm.bing.net/th?id=OIP.OJ6bilRfexVAdYyCtv0kMAHaBf&pid=Api
&P=0&w=569&h=115
Title: Addition and Subtraction of Radical Expressions
Link: https://www.cliffsnotes.com/assets/255866.png
Link: https://www.algebra-answer.com/tutorials-2/greatest-common-
factor/articles_imgs/3391/adding15.jpg
Link: http://saylordotorg.github.io/text_elementary-
algebra/section_11/fff2836446b2e64edb2ff111277679ab.jpg
Link: https://saylordotorg.github.io/text_elementary-
algebra/section_11/955d8066ed422f6995bffd3489fefb74.jpg

61
Green Rose Center for Academe Inc.
Purok Gemelina Estaca Compostela Cebu
Mobile No. 0933-1617936 / Landline No. (032) 425-6216
E-mail add: greenroseacademe_119@yahoo.com, Website: http://grca.school
Government Recognition No:
PRE-ELEM: 04 S. 2017, ELEMENTARY: 5, S. 2017, JHS: 06, S.2017, SHS: 059 S.2018
School ID: 408281
MATHEMATICS 9- DRILL 17
(Topic: Performing Addition and Subtraction of Radicals Expression)
Name: _____________________________Gr. & Sec. _________Date: ________
Parent’s Signature: ________________ Score: ________________
A. Directions: Find the sum and difference. Write your solution and answer
on the space being provided. (Note: 2 points for each number)
1. √2+√2 + 2√2 2. 17√11𝑦 + √11𝑦 − 53√11𝑦

3.3√3𝑎𝑏 + √3𝑎𝑏 + 5√3𝑎𝑏 4. −13√𝑚𝑛 − 5√𝑚𝑛 − 6√𝑚𝑛

5.9√4𝑝2 − √4𝑝2 + 5 √4𝑝2 6. √6 + √6 + 5√6


3 3 3

7. √𝑝2 − 17√𝑝2 + 53√𝑝2 8. 13√3𝑎𝑏𝑐 + √3𝑎𝑐𝑏 + 52√3𝑎𝑐𝑏


3 3 3

9. √7𝑥 + 4√7𝑥 − 6√7𝑥 10. 19 2√21𝑥𝑦 − 11 2√21𝑥𝑦 − 7 2√21𝑥𝑦

62
Green Rose Center for Academe Inc.
Purok Gemelina Estaca Compostela Cebu
Mobile No. 0933-1617936 / Landline No. (032) 425-6216
E-mail add: greenroseacademe_119@yahoo.com, Website: http://grca.school
Government Recognition No:
PRE-ELEM: 04 S. 2017, ELEMENTARY: 5, S. 2017, JHS: 06, S.2017, SHS: 059 S.2018
School ID: 408281
MATHEMATICS 9- ASSIGNMENT 17
(Topic: Performing Addition and Subtraction of Radicals Expression)

Name: __________________________Gr. & Sec. ____________Date: ________


Parent’s Signature: ________________ Score: ________________
A. Directions: Solve the word problem and express your answer as a radical
in simplest form. Write your solution and answer on the space being
provided.

1. If each leg of an isosceles triangle is three times the base and the
perimeter is 7√224 + 14 𝑐𝑚, find the length of the legs and base.

63
MATHEMATICS 9- LESSON 18
TOPIC: PERFORMING MULTIPLICATION OF RADICAL
EXPRESSIONS

To simplify expressions involving products of radical, you can use this property:
Product of Roots
If a and b are real numbers and a ≥ 0 and b ≥ 0, then
𝑛 𝑛
√𝑎 × √𝑏
𝑛
= √𝑎 × 𝑏
𝑛
= √𝑎𝑏
In general, the product of the nth root of two or more real numbers is the nth root
of the product of the numbers.
Example 1:

Example 2:

64
Example 3:

Example 4:

Example 5:

65
Example 6:

LIST OF LINKS
Title: Multiply the Radicals
Link: https://www.onlinemathlearning.com/image-files/multiply-radicals.png
Title: Multiplication is Commutative
Link: https://saylordotorg.github.io/text_elementary-
algebra/section_11/bd1022000e5d4a00a15ef9de1219bace.jpg
Title: Multiplication of Radicals
Link: https://www.odiji.com/wp-content/uploads/2020/08/adding-and-
subtracting-radicals-worksheet-multiplying-radical-expressions-of-adding-and-
subtracting-radicals-worksheet.jpg
Title: Multiplication of Coefficients
Link: http://saylordotorg.github.io/text_elementary-
algebra/section_11/01793dc64d2dd7a5cef3b8e16065b80d.jpg
Title: Multiplication of regular numbers
Link: https://www.katesmathlessons.com/uploads/1/6/1/0/1610286/multiplying-
two-radical-expressions_orig.png
Title: Multiplying Radicals
Link: http://www.cliffsnotes.com/assets/255934.png

66
Green Rose Center for Academe Inc.
Purok Gemelina Estaca Compostela Cebu
Mobile No. 0933-1617936 / Landline No. (032) 425-6216
E-mail add: greenroseacademe_119@yahoo.com, Website: http://grca.school
Government Recognition No:
PRE-ELEM: 04 S. 2017, ELEMENTARY: 5, S. 2017, JHS: 06, S.2017, SHS: 059 S.2018
School ID: 408281
MATHEMATICS 9- DRILL 18
(Topic: Performing Multiplication of Radicals Expression)
Name: _____________________________Gr. & Sec. _________Date: ________
Parent’s Signature: ________________ Score: ________________
A. Directions: Multiply the following radicals then simplify. Write your
solution and answer on the space being provided.
1. √5(√8) 2. √3(√7)

3. √10(√6) 4. 3√5(−4√2)

5.−6√10(44 √8)

B. Directions: Simplify each expression. Write your solution and answer on


the space being provided.

1. (4√𝑥 2 𝑦) (3√𝑥 2 𝑦) 2. (√3𝑚) (√2𝑚)(2√6𝑚)

3.(−3√𝑚2 𝑛) (4√𝑛2 𝑚) 4. (27)(√3𝑎)(√5𝑎)

5.(5√𝑎𝑏2 )(−2√𝑏) 6. (x  )(x  x )

7.(k )( k ) 8. (zz) 16)

9. x (x ) 10. 𝑟 4 (4𝑟 3 )

67
MATHEMATICS 9- LESSON 19
TOPIC: PERFORMING DIVISION OF RADICAL EXPRESSIONS

Quotient of Roots
𝒏
√𝒂 𝒏 𝒂
If a and b are real numbers and a ≥ 0 and b ≥ 0, then 𝒏 = √𝒃
√𝒃

√108
Example 1: Simplify
√12

Solution:
√108
√12

108
= √ 12

= √9
=3
3
√1250
Example 2: Simplify 3
√10

Solution∶
3
√1250
3
√10

3 1250
=√ 10

3
= √1253
=5
√195
Example 3: Simplify
√15

√195
√15

195
=√ 15

= √𝟏𝟑

68
Rationalizing Denominator
To rationalize the denominator means to eliminate any radical expressions in the
denominator such as square roots and cube roots. The key idea is to multiply the
original fraction by an appropriate value, such that after simplification, the
denominator no longer contains radicals.

√75
Example 4: Simplify
√2

√75
√2

√75 √2
= ×
√2 √2

√150
=
√4

√25×6
=
√4

𝟓√𝟔
= 𝟐

Example 5:

69
Example 6:

Example 7:

Example 8:

Example 9:

70
Example 10:

LIST OF LINKS
Title: Rationalizing Denominator
Link: https://knoji.com/images/user/formulasforDenominatorofaFraction-2.jpg
Title: Sample of Rationalizing Denominator
Link: https://saylordotorg.github.io/text_elementary-
algebra/section_11/f8840a593de17780d3f085d8c11c7c6b.jpg
Link: https://saylordotorg.github.io/text_elementary-
algebra/section_12/42767faea5b6a4e417d7fa84d876fe86.jpg
Link: https://saylordotorg.github.io/text_elementary-
algebra/section_11/4a69594e69ac3acdb2bd7eabc45878d0.jpg
Link: https://www.katesmathlessons.com/uploads/1/6/1/0/1610286/dividing-
radical-expressions-with-variables_orig.png
Link: http://s3.amazonaws.com/new-ea-copy/public-
web/section_11/5a3e2bae9fe4b98b1c640228279c904d.jpg
Link: http://saylordotorg.github.io/text_elementary-
algebra/section_11/a9d541b8f6b1ca5eb6f7e9fd3ebb5079.jpg

71
Green Rose Center for Academe Inc.
Purok Gemelina Estaca Compostela Cebu
Mobile No. 0933-1617936 / Landline No. (032) 425-6216
E-mail add: greenroseacademe_119@yahoo.com, Website: http://grca.school
Government Recognition No:
PRE-ELEM: 04 S. 2017, ELEMENTARY: 5, S. 2017, JHS: 06, S.2017, SHS: 059 S.2018
School ID: 408281
MATHEMATICS 9- DRILL 19
(Topic: Performing Division of Radicals Expression)
Name: _____________________________Gr. & Sec. _________Date: ________
Parent’s Signature: ________________ Score: ________________
A. Directions: Simplify the following. Write your solution and answer on the
space being provided.
√75 √320
1. 2.
√4 √100

9 49
3.√16 4. √81

252
5. √ 9

B. Directions: Simplify each expression by rationalizing the denominator.


Write your solution and answer on the space being provided.
√75 −3√5
1. 2.
√3 √7

2√5+√5 2−√5+√5
3. 4. −
√5 √3

√𝑛 √𝑛
5. 6.
√𝑚 √5+√𝑛

5+√3 5−√5+√3
7. 8.
√6+√7 √2+5

−√2−√18+√8 √2−√10+√8
9. 10.
√3 √5

72
Green Rose Center for Academe Inc.
Purok Gemelina Estaca Compostela Cebu
Mobile No. 0933-1617936 / Landline No. (032) 425-6216
E-mail add: greenroseacademe_119@yahoo.com, Website: http://grca.school
Government Recognition No:
PRE-ELEM: 04 S. 2017, ELEMENTARY: 5, S. 2017, JHS: 06, S.2017, SHS: 059 S.2018
School ID: 408281
MATHEMATICS 9- ASSIGNMENT 19
(Topic: Performing Division of Radicals Expression)

Name: __________________________Gr. & Sec. ____________Date: ________


Parent’s Signature: ________________ Score: ________________
A. Directions: Solve the word problem and express your answer as a radical
in simplest form. Write your solution and answer on the space being
provided.
1. How can you rationalize a denominator? Give examples.

73
MATHEMATICS 9- LESSON 20
TOPIC: SOLVING EQUATION INVOLVING RADICAL EXPRESSION
Radical Equation – an equation with a radical expression; the radicand of the
expression contains a variable.

Example: The equation √𝑥 − 5 = 8 is a radical expression.


Extraneous Root – a root of a transformed equation that is not a root of the
original equation.

Example: Solve for the solution of √𝑥 = −2.

√𝑥 = −2
(√𝑥)2 = −2
𝑥=2
It appears that 𝑥 = 4 is the solution of the equation. However, if we substitute 𝑥
as 4, we will get √4 = −2, which is false. Thus, 𝑥 = 4 is an extraneous root.
Solving Equations Involving Radical Expressions
In solving an equation involving a radical term, the general strategy is to isolate
the radical term on one side of the equation and then raise both sides to a power
so that the radical will be removed.

Example 1: Solve .

Isolate the radical expression.

Raise both sides to the index of the radical; in this case, square both sides.

This quadratic equation now can be solved either by factoring or by applying the
quadratic formula.

74
Applying the quadratic formula,

Now, check the results.

If ,

If x = –5,

The solution is or x = –5.


75
Example 2: Solve .

Isolate the radical expression.

There is no solution, since cannot have a negative value.

Example 3: Solve .

Isolate one of the radical expressions.

Raise both sides to the index of the radical; in this case, square both sides.

This is still a radical equation. Isolate the radical expression.

Raise both sides to the index of the radical; in this case, square both sides.

76
This can be solved either by factoring or by applying the quadratic formula.

Applying the quadratic formula,

Check the solutions.

If x = 10,

So x = 10 is not a solution.

If x = 2,

The only solution is x = 2.

LIST OF LINK:

Title: Solving Equations Involving Radical Expressions


Link: https://www.cliffsnotes.com/study-guides/algebra/algebra-ii/quadratics-in-
one-variable/solving-radical-equations

77
Green Rose Center for Academe Inc.
Purok Gemelina Estaca Compostela Cebu
Mobile No. 0933-1617936 / Landline No. (032) 425-6216
E-mail add: greenroseacademe_119@yahoo.com, Website: http://grca.school
Government Recognition No:
PRE-ELEM: 04 S. 2017, ELEMENTARY: 5, S. 2017, JHS: 06, S.2017, SHS: 059 S.2018
School ID: 408281
MATHEMATICS 9- DRILL 20
(Topic: Solving Equation Involving Radical Expression)
Name: _____________________________Gr. & Sec. _________Date: ________
Parent’s Signature: ________________ Score: ________________
A. Directions: Solve each equation and check the solution. Write your
solution clearly.
(Note: 1 point for solving equation and 1 point for checking the solution)
1. √𝑥 − 5 = 4 2. √𝑥 + 4𝑥 = −4

3.4√7 − 5 = √𝑎 4. √9𝑥𝑦 − 2𝑥 = √𝑦

5.√𝑏 − 1 = 𝑏 − 3 6. -√𝑚 + 1 = 𝑚 − 3𝑛 + 4𝑚

3 3 1
7.5 + √𝑛 = 1 8. 5 + √𝑛 = 1 − 𝑛

𝑘 𝑘
9.√2 − 7 = 4 10. √2 − 7 = 4 +√𝑘

78
MATHEMATICS 9- LESSON 21
TOPIC: PROVING PYTHAGOREAN THEOREM

Theorem 5.7.1 (The Pythagorean Theorem)


In a right triangle, the square of the
hypotenuse is equal to the sum of the
squares of the legs.
The longest side of the triangle is
called the "hypotenuse", so the
formal definition is:
In a right triangle, the square of the hypotenuse is equal to the sum of the
squares of the other two sides.
Formula:
c2 = a2 + b2
Example 1:

Start with: a2 + b2 = c2
Put in what we know: 52 + 122 = c2
Calculate squares: 25 + 144 = c2
25+144=169: 169 = c2
Swap sides: c2 = 169

Square root of both sides: c = √169


Calculate: c = 13
Theorem 5.7.2 (The Converse of Pythagorean Theorem)
If the square of one side of a triangle is equal to the sum of the squares of the
other two sides, then the triangle is a right triangle.

79
Theorem 5.7.3
If a<b<c are lengths of the sides of a triangle then,
a2 + b2 = c2, then the triangle is a right triangle.
a2 + b2 < c2, then the triangle is an obtuse triangle.
a2 + b2 > c2, then the triangle is an acute triangle.
Example 1:
Triangle with the sides 5, 6, and 8. Is it acute, right, or obtuse?
Solution:
5 2 + 6 2? 8 2
25 + 36? 64
61 < 64 (Obtuse Triangle)

LIST OF LINKS
Title: Pythagorean Theorem
Link: http://2.bp.blogspot.com/-
jmfnlMccOCM/UqaXQM73I2I/AAAAAAAAAy8/lmZp97DCKxY/s1600/rt+tri.png
Title: Example of Pythagorean Theorem
Link: https://www.mathsisfun.com/geometry/images/triangle-5-12-c.svg

80
Green Rose Center for Academe Inc.
Purok Gemelina Estaca Compostela Cebu
Mobile No. 0933-1617936 / Landline No. (032) 425-6216
E-mail add: greenroseacademe_119@yahoo.com, Website: http://grca.school
Government Recognition No:
PRE-ELEM: 04 S. 2017, ELEMENTARY: 5, S. 2017, JHS: 06, S.2017, SHS: 059 S.2018
School ID: 408281
MATHEMATICS 9- DRILL 21
(Topic: Proving Pythagorean Theorem)
Name: _____________________________Gr. & Sec. _________Date: ________
Parent’s Signature: ________________ Score: ________________
A. Directions: Determine whether a triangle with sides of the given lengths
is acute, right or obtuse. Write your answer on the space being provided.
_________1. 11, 12, 15
_________2. 8, 14, 17
_________3. 15, 20, 25
_________4. 33, 44, 55
_________5. 5, 12, 13
B. Directions: Find each missing length of a right triangle, c is length of
hypotenuse. Write your answer on the space being provided.
1. If a=9, b=12, c=_______.

2. If a=√3, b=√5, c=_______.


3. If c=17, a=8, b=_________.
4. If a=3, b=3, c=__________.

5. If a=2√3, c=6, b=________.


C. Directions: Find the value of 𝑥. Write your solution and answer on the
space being provided.
1.
13
x

12

81
2.
17
x

15

3.
x
6

4.
26
x

20

5.
x

5√3

MATHEMATICS 9- ASSIGNMENT 21
(Topic: Proving Pythagorean Theorem)

Name: __________________________Gr. & Sec. ____________Date: ________


Parent’s Signature: ________________ Score: ________________
A. Directions: Solve the word problem. Write your solution and answer on
the space being provided.
1. Find the length of the hypotenuse of a right triangle whose legs measure
9 and 12.

82
MATHEMATICS 9- LESSON 22
TOPIC: PROVING SSS, SAS, & ASA Postulate

Two triangles are congruent if their corresponding sides are equal in length and
their corresponding interior angles are equal in measure.
Corresponding sides and angles mean that the side on one triangle and the side
on the other triangle, in the same position, match. You may have to rotate one
triangle, to make a careful comparison and find corresponding parts.

Postulate 5.4.1 (AAA Similarity Postulate)


If three angles of one triangle are congruent to three angles of another
triangle, then the two triangles are similar.

Corollary T5.4.1-1
Two right triangles are similar if an acute angle of one triangle is
congruent to an acute angle of the second triangle.

83
SSS Theorem (Side-Side-Side)
Side Side Side Postulate (SSS) says triangles are congruent if three sides of one
triangle are congruent to the corresponding sides of the other triangle.

Introducing a diagonal into any of those shapes creates two triangles. Using any
postulate, you will find that the two created triangles are always congruent.
Suppose you have parallelogram SWAN and add diagonal SA. You now have two
triangles, △ SAN and △ SWA.

Line SA used in both triangles, is congruent to itself. ∠ SAN is congruent to


∠ WSA because they are alternate interior angles of the parallel line segments
SW and NA (because of the Alternate Interior Angles Theorem)
Line segments SW and NA are congruent, because they were part of the
parallelogram (opposite sides are parallel and congruent).
side SA, an included angle ∠ WSA, and a side SW of △ SWA.You can compare
those three triangle parts to the corresponding parts of △ SAN:
>Side SA≅Side SA
>Included angle ∠ WSA≅∠ NAS
>Side SW≅Side NA

84
SAS Theorem (Side-Angle-Side)
By applying the Side Angle Side Postulate (SAS), you can also be sure your two
triangles are congruent. Here, instead of picking two angles, we pick a side and
its corresponding side on two triangles.

The SAS Postulate says that triangles are congruent if any pair of
corresponding sides and their included angle are congruent.

For the two triangles to be congruent, those three parts -- a side, an included
angle, and adjacent side -- must be congruent to the same three parts -- the
corresponding side, angle and side -- on the other triangle, △ YAK.

ASA Theorem (Angle-Side-Angle)


The Angle Side Angle Postulate (ASA) says triangles are congruent if any two
angles and their included side are equal in the triangles. An included side is the
side between two angles.

85
Hypotenuse Leg Theorem
The hypotenuse leg theorem states that any two right triangles that have a
congruent hypotenuse and a corresponding, congruent leg are congruent
triangles.

LIST OF LINKS
Title: Triangles

Link: https://cdn.tutors.com/assets/images/courses/math/geometry-help/tutors-triangle-congruence-definitions-similar-
congruent.jpg

Title: AAA Similarity Postulate

Link: https://us-static.z-dn.net/files/d1f/ee117cfcf5a89f1597fac2c92e2caceb.jpg

Title: Corollary

Link: https://dr282zn36sxxg.cloudfront.net/datastreams/f-
d%3A941acc5da8d73e9d817c4459aa5a87e334bfafbf288266fec44fd822%2BIMAGE_TINY%2BIMAGE_TINY.1

Title: Triangle Congruence

Link: https://image.slidesharecdn.com/trianglecongruenceintro-170615135542/95/triangle-congruence-introduction-4-
638.jpg?cb=1497535156

Title: Three Postulates

Link: https://cdn.tutors.com/assets/images/courses/math/geometry-help/tutors-triangle-congruence-postulates-and-
theorems.jpg

Title: SSS Theorem

Link: https://cdn.tutors.com/assets/images/courses/math/geometry-help/tutors-sss-theorem-triangle-congruence.jpg

Title: Parallelogram

Link: https://cdn.tutors.com/assets/images/courses/math/geometry-help/congruence-polygons.jpg

Title: SAS Postulate

Link: https://cdn.tutors.com/assets/images/courses/math/geometry-help/tutors-sas-theorem-triangle-congruence.jpg

Title: ASA Theorem

Link: https://cdn.tutors.com/assets/images/courses/math/geometry-help/tutors-asa-theorem-triangle-congruence.jpg

Title: Hypotenuse Leg Theorem

Link:https://www.mathwarehouse.com/geometry/congruent_triangles/images/hypotenuse-leg/hypotenuse-leg-picture-
labelled-small.webp

86
Green Rose Center for Academe Inc.
Purok Gemelina Estaca Compostela Cebu
Mobile No. 0933-1617936 / Landline No. (032) 425-6216
E-mail add: greenroseacademe_119@yahoo.com, Website: http://grca.school
Government Recognition No:
PRE-ELEM: 04 S. 2017, ELEMENTARY: 5, S. 2017, JHS: 06, S.2017, SHS: 059 S.2018
School ID: 408281
MATHEMATICS 9- DRILL 22A
(Topic: Proving SSS, SAS, & ASA Postulate)
Name: _____________________________Gr. & Sec. _________Date: ________
Parent’s Signature: ________________ Score: ________________
A. Directions: Write TRUE if the statement is correct and FALSE if it is not.
Write your answer on the space being provided.
_____________1. If three angles of one triangle are congruent to three angles
of another triangle, then the two triangles are similar.
_____________2. The Angle Side Angle Postulate says triangles are congruent if
any two angles and their included side are equal in the triangles.
_____________3. Two right triangles are similar if an acute angle of one triangle
is congruent to an acute angle of the second triangle is called Triangle Postulate.
_____________4. The ASA Postulate says that triangles are congruent if any pair
of corresponding sides and their included angle are congruent.
_____________5. The SSS Postulate says triangles are congruent if three sides
of one triangle are congruent to the corresponding sides of the other triangle.
B. Direction: Identify the theorem.
1. Two right triangles that have a congruent hypotenuse and a corresponding
congruent leg are congruent.
2. If two angles and the non-included side of one triangle are congruent to
the corresponding parts of another triangle, the triangles are congruent.
3. If two angles and the included side of a triangle are congruent to two
angles and the included side of another triangle, then the two triangles are
congruent.
4. If two sides and the included angle of a triangle are congruent to two sides
and the included angle of another triangle, then the two triangles are congruent.
5. If three sides of one triangle are congruent to three sides of another
triangle, then the two triangles are congruent.

87
Green Rose Center for Academe Inc.

Purok Gemelina Estaca Compostela Cebu


Mobile No. 0933-1617936 / Landline No. (032) 425-6216
E-mail add: greenroseacademe_119@yahoo.com, Website: http://grca.school
Government Recognition No:
PRE-ELEM: 04 S. 2017, ELEMENTARY: 5, S. 2017, JHS: 06, S.2017, SHS: 059 S.2018
School ID: 408281
MATHEMATICS 9- DRILL 20B
(Topic: Proving SSS, SAS, & ASA Postulate)
Name: _____________________________Gr. & Sec. _________Date: ________
Parent’s Signature: ________________ Score: ________________
C. Directions: Complete each congruence statement by naming the
corresponding angle or side. Write your answer on the space being
provided.
1. Line FD is congruent to line ______. 2. Angle M is congruent to angle ____.

3.Angle U is congruent to angle _____. 4. Angle W is congruent to angle___.

5.Angle F is congruent to angle _______.

88
MATHEMATICS 9- LESSON 23
TOPIC: PROVING ANGLES IN TRIANGLES

In a triangle, there exists a relationship among its part. The following theorems
and corollaries deal with the way angles of a triangle are related.
Triangle Sum Theorem
The sum of the measures of the angles of a triangle is 180 degrees.
Corollaries:
1. Each angle of an equiangular triangle has a measure of 60 degrees.
2. In a triangle, there can be at most one right angle or obtuse angle.
3. The acute angles of a right triangle are complementary.
Example 1:
Find the measure of the angle.

Solution:
m∠M + m∠A + m∠T = 180
82 + 27 + m∠T = 180
m∠T = 180 - 82 - 27
m∠T = 98 - 27
m∠T = 71
Exterior and Interior Angle of a Triangle

89
Definition:
1. An exterior angle of a triangle is an angle which is adjacent and
supplementary to one of the angles of a triangle.
2. The remote interior angles of a triangle are angles which are not
adjacent to the exterior angle of a triangle.
3. An adjacent interior angle of a triangle is an interior angle which
forms a linear pair with an exterior angle.
Exterior Angle Equality Theorem

LIST OF LINKS:
Title: Measure of Angles
Link: http://cimg1.ck12.org/datastreams/f-
d%3A5503cadb5861929de383ed44b584fe9135b3312d4c3236aa1366a2bf%2BIM
AGE%2BIMAGE.1
Link: https://www.basic-mathematics.com/images/angle-sum-theorem.gif
Title: Exterior and Interior Angle of a Triangle
Link: http://www.onlinemathlearning.com/image-files/exterior-angles-
triangle.png
Title: Exterior Angle Equality Theorem
Link:http://passyworldofmathematics.com/Images/pwmImagesFour/TriangleExte
riorAngleThree540x482JPG.jpg

90
Green Rose Center for Academe Inc.
Purok Gemelina Estaca Compostela Cebu
Mobile No. 0933-1617936 / Landline No. (032) 425-6216
E-mail add: greenroseacademe_119@yahoo.com, Website: http://grca.school
Government Recognition No:
PRE-ELEM: 04 S. 2017, ELEMENTARY: 5, S. 2017, JHS: 06, S.2017, SHS: 059 S.2018
School ID: 408281
MATHEMATICS 9- DRILL 23
(Topic: Proving Angles in Triangles)
Name: _____________________________Gr. & Sec. _________Date: ________
Parent’s Signature: ________________ Score: ________________
A. Directions: Solve the unknown problem. Write your solution and
answer on the space being provided. (Note: 2 point for every number)

114°

x° 47°

1. Find the value of x.

38°

19°

56° x° z° y°

2. Find the measure of angle x.


3. Find the measure of angle z.
4. Find the measure of angle y.
5. Find the total measure of angle x and y.
B. Directions: Use <, > or = to compare the measure of an angles. Write
your answer on the space being provided.

38°

19°

56° x° z° y°

1. ∠x________38° 2. ∠x________58°

91
3. ∠z________38° 4. ∠y________56°
5. ∠z________26° + 70° 6. ∠x + ∠y________180°
7. ∠x________∠y 8. ∠x − ∠y________53°
9. 67° + 23°________∠z 10. ∠x + ∠z_______238°
MATHEMATICS 9- ASSIGNMENT 23
(Topic: Proving Angles of Triangles)

Name: __________________________Gr. & Sec. ____________Date: ________


Parent’s Signature: ________________ Score: ________________
A. Directions: Solve for x. Write your solution and answer on the space being
provided.

𝑥 + 41°

80° 35°

92
MATHEMATICS 9- LESSON 24
TOPIC: ILLUSTRATING UNIT CIRCLE

A circle is in standard position if and only if it lies in the coordinate plane


and its center is at origin. If the radius of a circle in standard position measures
one unit, then it is called a unit circle. Using Pythagorean Theorem, you can
derive the equation of the unit circle, then x2 + y2 = 1.
Every point on the unit of the circle is called trigonometric point.

Determining Unit Circle


Example 1: Determine whether the point (1/2, √3/2) is a point on the unit circle.
Solution:
To determine whether the point is a trigonometric point, it must satisfy the
equation x2 + y2 = 1.
x 2 + y2 = 1
(1/2)2 + (√3/2)2 = 1
1/4 + 3/4 = 1
4/4 = 1
1=1
Hence, the point (1/2, √3/2) is a point on the unit circle.

93
Example 2: Determine whether the point (-1/4, -3/4) is a point on the unit circle.
Solution:
x 2 + y2 = 1
(-1/4)2 + (-3/4)2 = 1
1/16 + 9/16 =1
10/16 = 1
5/8 = 1
Hence, the point (-1/4, -3/4) is not a point on the unit circle.

Convert Radian Measure to Degree Measure


Example 1: Convert π/6 to degree measure.
= (π/6)(180°/π)
= 180°/6
= 30°

Convert Degree Measure to Radian Measure


Example 1: Convert 25° to radian measure.
= (25°)(π/180°)
= 25π/180°
= 5π/36

LIST OF LINK
Title: Unit Circle
Link: https://36gu5d4dxary1824ba1o7kkq6uc-wpengine.netdna-ssl.com/help/wp-
content/uploads/sites/2/2014/12/unit-circle-with-ratios-of-pi.png

94
Green Rose Center for Academe Inc.
Purok Gemelina Estaca Compostela Cebu
Mobile No. 0933-1617936 / Landline No. (032) 425-6216
E-mail add: greenroseacademe_119@yahoo.com, Website: http://grca.school
Government Recognition No:
PRE-ELEM: 04 S. 2017, ELEMENTARY: 5, S. 2017, JHS: 06, S.2017, SHS: 059 S.2018
School ID: 408281
MATHEMATICS 9- DRILL 24
(Topic: Illustrating Unit Circle)
Name: _____________________________Gr. & Sec. _________Date: ________
Parent’s Signature: ________________ Score: _________________
A. Directions: Fill the blanks. Write your answer on the space being
provided.

B. Directions: Convert the following into radian to degree measure and vice
versa. Write your answer on the space being provided.
𝜋 4𝜋 2𝜋
1. 2. 3.
5 3 4

4.270° 5. 160°

95
MATHEMATICS 9- LESSON 25
TOPIC: ILLUSTRATING ANGLES IN STANDARD POSITION

An angle can be generated by rotating ray BA about its endpoint B to the


position of ray BC. Ray BC is called the initial side of the angle, ray BA the
terminal side and B the vertex.

You often represent angles by Greek letter such as θ (theta), α (alpha), β


(beta). If an angle is generated by counterclockwise rotation, then θ is a
positive angle. When the rotation is clockwise, θ is a negative angle. Arrows
are used to indicate the direction or rotation.

An angle θ is said to be in standard position when its vertex is at the origin of a


rectangular coordinate plane and its initial side lies on the positive horizontal
axis.

96
If the terminal side of θ lies in Quadrant 1, then θ is called a first-quadrant
angle. Similarly, there are second-quadrant angle, third-quadrant angle,
and fourth-quadrant angle. An angle whose terminal side lies on an axis is
called a quadrantal angles.

Complementary and Supplementary Angles


Two positive angles A and B are complementary if their sum is 90°. Two positive
angles are supplementary if their sum is 180°.

Example 1: What is the complement of each angle given the following


measures?
1. 78°
= 90° - 78°
= 12°

2. 25°14’
= 90° - 25°14’
= 89°60’ - 25°14’
= 64°46’

3. 36°12’48”
= 90° - 36°12’48”
= 89°59’60” - 36°12’48”
= 53°47’12”

97
Example 2: What is the supplement of each angle given the following
measures?
1. 105°38”
= 180° - 105°38”
= 179°59’60” - 105°38”
= 74°59’22”

2. 47°42’23”
= 180° - 47°42’23”
= 179°50’60” - 47°42’23”
= 132°17’37”
Example 3: Convert 68°15’40” to decimal form.
68°15’40”
= 68° + (15/60)° + (403600)°
= 68° + 0.25° + 0.0111°
= 68.2611°

LIST OF LINKS
Title: Angles in Standard Position
Link: https://www.mathopenref.com/images/angles/partnames.png
Title: Positive and Negative Angles
Link: http://www.mathsisfun.com/geometry/images/positive-negative-angle.gif
Title: Angles
Link: https://i.ytimg.com/vi/IyPdK4_DYdQ/hqdefault.jpg
Title: Quadrantal Angles
Link: https://edudelighttutors.com/wp-content/uploads/2020/02/image-result-
for-quadrant-and-angles.png

98
Green Rose Center for Academe Inc.
Purok Gemelina Estaca Compostela Cebu
Mobile No. 0933-1617936 / Landline No. (032) 425-6216
E-mail add: greenroseacademe_119@yahoo.com, Website: http://grca.school
Government Recognition No:
PRE-ELEM: 04 S. 2017, ELEMENTARY: 5, S. 2017, JHS: 06, S.2017, SHS: 059 S.2018
School ID: 408281
MATHEMATICS 9- DRILL 25
(Topic: Illustrating Angles in Standard Position)
Name: _____________________________Gr. & Sec. _________Date: ________
Parent’s Signature: ________________ Score: ________________
A. Directions: Do as directed. Write your answer on the space being
provided.
1. Sketch an angle of 40°in standard position.

2. Sketch an angle of −155°in standard position.

3. Sketch an angle of 35°in standard position.

4. Sketch an angle of -35°in standard position.

5. Sketch an angle of 120°in standard position.

99
B. Directions: Indicate the quadrant of each of the following angles. Write
your answer on the space being provided.
1. 130°
2. −140°
7𝜋
3.
4
4. 𝜋
5.2𝜋

C. Directions: Do as directed. Write your answer on the space being


provided.
1. Find the supplement of each angle:
a. 30°
b. 81°
c. 36°
2. Find the complement of each angle:
𝑎. 30°
𝑏. 81°

MATHEMATICS 9- ASSIGNMENT 25
(Topic: Illustrating Angles in Standard Position)

Name: __________________________Gr. & Sec. ____________Date: ________


Parent’s Signature: ________________ Score: ________________
A. Directions: Do as directed. Write your solution and answer on the space
being provided.
1. Convert 78°15’40” to decimal form.

100
MATHEMATICS 9- LESSON 26
TOPIC: FINDING TRIGONOMETRIC RATIOS

Right Triangle – a triangle with one right angle or with one interior angle of
90°
Trigonometry – a branch of mathematics that deals with the relationships of the
lengths and the angles of triangles. Trigonometry comes from the Greek word
trigonon, which means “triangle,” and metron, which means “to measure.”

Parts of a Right Triangle


 Hypotenuse – the longest side of the triangle; it is always opposite to the
right angle
 Adjacent side – the side next to theta, θ, but not the hypotenuse
Example: The illustration below shows the adjacent side of the right
triangle
 Opposite side – the side opposite to theta, θ

The Six Trigonometric Ratios

In a right triangle, there are six possible trigonometric ratios as follows:


A

opposite hypotenuse

C adjacent B
opposite adjacent opposite
sin   cos   tan  
hypotenuse hypotenuse adjacent
hypotenuse hypotenuse adjacent
csc   sec  cot  
opposite adjacent opposite
For easier recall, use the mnemonic SOH CAH TOA & CHO SHA CAO

101
The Special Triangles
• The 𝟒𝟓° − 𝟒𝟓° − 𝟗𝟎° Triangle – an isosceles right triangle, whose two sides
are equal

Example: The triangle below is a 𝟒𝟓° − 𝟒𝟓° − 𝟗𝟎° Triangle.

• The 𝟑𝟎° − 𝟔𝟎° − 𝟗𝟎° Triangle – a right, scalene triangle whose ratio of
hypotenuse, opposite, and adjacent is 2: √3: 1. It can be derived from an
equilateral triangle with side length 2. Each of its internal angle measures
60°. Dividing the figure into two similar partitions, we arrive at two 30° −
60° − 90° triangle.

LIST OF LINK
Title: Special Right Triangle
Link:https://www.dummies.com/education/math/calculus/how-to-work-with-45-
45-90-degree-triangles/

102
Green Rose Center for Academe Inc.
Purok Gemelina Estaca Compostela Cebu
Mobile No. 0933-1617936 / Landline No. (032) 425-6216
E-mail add: greenroseacademe_119@yahoo.com, Website: http://grca.school
Government Recognition No:
PRE-ELEM: 04 S. 2017, ELEMENTARY: 5, S. 2017, JHS: 06, S.2017, SHS: 059 S.2018
School ID: 408281
MATHEMATICS 9- DRILL 26
(Topic: Finding Trigonometric Ratios)
Name: _____________________________Gr. & Sec. _________Date: ________
Parent’s Signature: ________________ Score: ________________
A. Directions: Do as directed. Write your answer on the space being
provided.

___________1. Find sin A __________2. Find sec B


___________3. Find sin C ___________4. Find tan A
___________5. Find tan C ___________6. Find cos A
___________7. Find cos B ___________8. Find cot A
___________9. Find csc B ___________10.Find sec C
B. Direction: Fill in the blanks.
1. The ratio of the length of the opposite side of an acute angle and its
adjacent side is _______________.
2. The ratio of the length of the _______________ side of an acute angle
and its opposite side is cotangent.
3. The ratio of the length of the adjacent side of an acute angle and its
hypotenuse side is _____________.
4. The ratio of the length of the hypotenuse side of an acute angle and its
adjacent side is _______________.
5. The ratio of the length of the hypotenuse side of an acute angle and its
opposite side is ____________.

103
MATHEMATICS 9- LESSON 27
TOPIC: SOLVING ANGLES OF ELEVATION

Definition of Angle of Elevation:

The angle of elevation of an object as seen by the observer is defined as


the angle between the horizontal and the line from the object to the observer’s
eye. The line in which observer’s eye is there is known as the line of sight.

Example 1:

A nursery plants a new tree and attaches a guy wire to help support the
tree while its roots take hold. An 8 feet wire is attached to the tree and to a
stake in the ground. From the stake in the ground the angle of elevation of the
connection with the tree is 42º. Find to the nearest tenth of a foot, the height of
the connection point on the tree.

104
Solution:
• A "guy" wire is a support wire used to hold a newly planted tree in place,
preventing it from bending or up-rooting during high winds.
• The "angle of elevation" is from the ground up.
• It is assumed that the tree is vertical, making it perpendicular with the ground.
• This problem deals with "opposite" and "hypotenuse" making it a sine problem.

Example 2:

Find the shadow cast by a 10 feet lamp post when the angle of elevation
of the sun is 58º. Find the length to the nearest tenth of a foot.

Solution:
• Remember that the "angle of elevation" is from the horizontal ground line
upward.
• It is assumed that the lamp post is vertical, making it perpendicular with the
ground.
• Shadows are on the ground! If you place the "shadow" on the hypotenuse you
have created an apparition ( a "ghost"), not a shadow!
• This solution deals with "opposite" and "adjacent" making it a tangent problem.

105
Example 3:

A radio station tower was built in two sections. From a point 87 feet from
the base of the tower, the angle of elevation of the top of the first section is 25º,
and the angle of elevation of the top of the second section is 40º. To the nearest
foot, what is the height of the top section of the tower?

Solution:
• Think of this problem as working with two separate triangles:
(1) the larger triangle with the 40º angle and a vertical side that represents the
ENTIRE height, b, of the tower, and
(2) the smaller triangle with the 25º angle and a vertical side, a, that represents
the height of the first (bottom) section of the tower.
• Solve for the vertical heights (b and a) in the two separate triangles.
• The needed height, x, of the second (top) section of the tower will be the
difference between the ENTIRE height, b, and the height of the first (bottom)
section, a. You will need to subtract.
• In both triangles, the solution deals with "opposite" and "adjacent" making it a
tangent problem.
• Larger triangle with

height b:
• Smaller triangle with

height a:

• Difference (b - a): 73.00166791 - 40.56876626 = 32.43290165 ≈ 32 feet

LIST OF LINK
Title: All Pictures of Angle of Elevation

Link:https://mathbitsnotebook.com/Geometry/Trigonometry/TGElevDepress.html

106
Green Rose Center for Academe Inc.
Purok Gemelina Estaca Compostela Cebu
Mobile No. 0933-1617936 / Landline No. (032) 425-6216
E-mail add: greenroseacademe_119@yahoo.com, Website: http://grca.school
Government Recognition No:
PRE-ELEM: 04 S. 2017, ELEMENTARY: 5, S. 2017, JHS: 06, S.2017, SHS: 059 S.2018
School ID: 408281
MATHEMATICS 9- DRILL 27
(Topic: Solving Angles of Elevation)
Name: _____________________________Gr. & Sec. _________Date: ________
Parent’s Signature: ________________ Score: ________________
A. Directions: Solve the word problem. Write your solution and answer on
the space being provided. (Note: 5 points for each number)
1. You are measuring the height of a lamp post. You stand 40 inches from
the base of the lamp post. You measure the angle of elevation from the
ground to the top of the lamp post to be 70°. Find the height ℎ of the
lamp post to the nearest inch.

2. A 10-ft flagpole casts a shadow of 18 ft. Find the angle of elevation of the
sun. Round your answer to the nearest degree.

3. Find the length of the shadow casted by a 10-foot lamp post when the
angle of elevation of the sun is 58°. Express the length to the nearest
tenth of a foot.

107
4. At a certain point 62 m from the base of your mom’s office building, the
angle of elevation of the top of the building is 70°. How tall is the building
to the nearest hundredth?

MATHEMATICS 9- ASSIGNMENT 27
(Topic: Solving Angle of Elevation)

Name: __________________________Gr. & Sec. ____________Date: ________


Parent’s Signature: ________________ Score: ________________
A. Directions: Solve the word problem. Write your solution and answer on the
space being provided.
1. A radio station tower was built in two sections. From a point 80 feet from
the base of the tower, the angle of elevation of the top of the first section
is 25°, while the angle of elevation of the top of the second section is 40°.
What is the height of the second section of the tower to the nearest foot?

108
MATHEMATICS 9- LESSON 28
TOPIC: SOLVING ANGLES OF DEPRESSION

Definition of Angle of Depression

The angle between the horizontal axis and the line of sight to the object
when the observer looks downward.

Example 1:

From the top of a fire tower, a forest ranger sees his partner on the
ground at an angle of depression of 40º. If the tower is 45 feet in height, how
far is the partner from the base of the tower, to the nearest tenth of a foot?

Solution:
• Remember that the "angle of depression" is from a horizontal line of sight
downward.
• It is assumed that the tower is vertical, making it perpendicular with the
ground.
• This solution will use alternate interior angles from the parallel horizontal lines,
so place 40º inside the triangle by the partner (bottom right).
• This solution deals with "opposite" and "adjacent" making it a tangent problem.

109
Example 2:

A ladder leans against a brick wall. The foot of the ladder is 6 feet from
the wall. The ladder reaches a height of 15 feet on the wall. Find to the nearest
degree, the angle the ladder makes with the wall.

Solution:
• In this problem place xº where the ladder meets the wall. Do not assume that
the angle will always be at the ground level.
• It is assumed that the wall is vertical, perpendicular with the ground.
• The foot of the ladder is the bottom of the ladder, where it hits the ground.
• This solution deals with "opposite" and "adjacent" making it a tangent problem.

Example 3:

From the top of a building, the angle of depression of the top and the foot of a
lamp post are 30° and 60° respectively. What is the height of the lamp post?

Solution:

According to the problem, the height of the building PQ = 12 m.

Let height of the lamp post RS.

Angle of depression of the top of a lamp post is 30°

Therefore, ∠TPR = 30°.

again, Angle of depression of the foot of a lamp post is 60°

Therefore, ∠TPS = 60°.

PQ = TS = 12 m.

Let the height of the lamp post RS = h m.


110
Therefore,

TR = (12 - h) m.

Also, let PT = x m

Now tan ∠TPR = TRPTTRPT = tan 30°


Therefore, 12−hx12−hx = 1√31√3 ........... (i)
Again, tan ∠TPS = TSPTTSPT = tan 60°
Therefore, 12x12x = √3 ........... (ii)

Dividing (i) by (ii), we get

12−h1212−h12 = 1313

⟹ 36 - 3h = 12

⟹ 3h = 36- 12

⟹ 3h = 24

⟹ h = 243243

⟹h=8

Therefore, height of the lamp post is 8 meters.

LIST OF LINKS
Title: Example 1 and 2 Angle of Depression

Link:https://mathbitsnotebook.com/Geometry/Trigonometry/TGElevDepress.html

Title: Example 3 Angle of Depression

Link: https://www.math-only-math.com/angle-of-depression.html#gallery[pageGallery]/6/

111
Green Rose Center for Academe Inc.
Purok Gemelina Estaca Compostela Cebu
Mobile No. 0933-1617936 / Landline No. (032) 425-6216
E-mail add: greenroseacademe_119@yahoo.com, Website: http://grca.school
Government Recognition No:
PRE-ELEM: 04 S. 2017, ELEMENTARY: 5, S. 2017, JHS: 06, S.2017, SHS: 059 S.2018
School ID: 408281
MATHEMATICS 9- DRILL 28
(Topic: Solving Angles of Depression)
Name: _____________________________Gr. & Sec. _________Date: ________
Parent’s Signature: ________________ Score: ________________
A. Directions: Solve the word problem. Write your solution and answer on
the space being provided. (Note: 5 points for each number)
1. The angle of depression from a lamp post to a point on the ground is 60°.
How long is the light beam to the nearest whole number if the lamp post
is 10 m high?

2. Bryan is standing on top of a hill and spots a house in the adjacent valley
at an angle of depression of 12°. The base of the hill to Bryan’s line of
sight is 60 m high. How far is the house from the base of the hill?

3. The top of the Bantayan light house is 127 meters above sea level. The
angle of depression from the top of the light house to the approaching
ship is 32°. What is the distance of the ship from the foot of the light
house?

4. A man standing on an elevated bridge sees two cars approaching him on


a road below the bridge. The angle of depression to the two cars from the
man’s line of sight are 15° and 40°, respectively. If the man’s eyes are 79
feet above the road where the two cars are, find the distance between the
two cars.

112

You might also like